Ask Our Doctors

Dear Patients,

I created this forum to welcome any questions you have on the topic of infertility, IVF, conception, testing, evaluation, or any related topics. I do my best to answer all questions in less than 24 hours. I know your question is important and, in many cases, I will answer within just a few hours. Thank you for taking the time to trust me with your concern.

– Geoffrey Sher, MD

Fill in the following information and we’ll get back to you.

Name: Carmen T

Cotización de insiminacion invitro o reconstrucción de trompas

Answer:

Please re-post in English!

 

Geoff Sher

IVF

Name: Mikayla h

This question is about reciprocal IVF. Im 21 and my wife is 58. She hasn’t reached menopause and I have irregular periods. I was diagnosed with infertility at a young age after a miscarriage. Who would you recommend to use the eggs and carry? We want to use my eggs and have her carry but unsure if it’ll be safe

Answer:

Youn could do so, but I must caution you that there would be significant medical risk in a woman of 58Y carrying a pregnancy.

 

Geoff Sher

++++++++++++++++++++++++++++++++++++++++++++++++

PLEASE SHARE THIS WITH OTHERS AND HELP SPREAD THE WORD!!

 

Herewith are  online links to 2  E-books recently  co-authored with  my partner at SFS-NY  (Drew Tortoriello MD)……. for your reading pleasure:

  1. From In Vitro Fertilization to Family: A Journey with Sher Fertility Solutions (SFS) ; https://sherfertilitysolutions.com/sher-fertility-solutions-ebook.pdf

 

  1. Recurrent Pregnancy Loss and Unexplained IVF Failure: The Immunologic Link ;https://drive.google.com/file/d/1iYKz-EkAjMqwMa1ZcufIloRdxnAfDH8L/view

 

I invite you to visit my very recently launched “Podcast”,  “HAVE A BABY” on RUMBLE;   https://rumble.com/c/c-3304480

If you are interested in having an online consultation with me, please contact my assistant, Patti Converse at 702-533-2691 or email her at concierge@sherivf.com\

 

Name: Sherry P

other Hi Dr. Sher,

I have two embryos frozen on day 3 (not day 5 for some reason). . One embryo is 8 cell grade 1 and the other is 10 cell grade 1. Are these good quality embryos? What are my chances of getting pregnant? I am in my mid 40s.

Thank you,

Sherry

Answer:

This is very much dependent on your age and the clinical underlying circumstances. While ordinarily anything better than a 5c day-3 embryo are positive feature3s, there are many other considerations. If you wanted better comfort, I would suggest these 2 embryos be thawed, carried to blastocyst and biopsied for PGT by no later than day 6. This would not in my opinion, prejudice the embryos.

 

Geoff Sher

__________________________________________________________________________

PLEASE SHARE THIS WITH OTHERS AND HELP SPREAD THE WORD!!

 

Herewith are  online links to 2  E-books recently  co-authored with  my partner at SFS-NY  (Drew Tortoriello MD)……. for your reading pleasure:

  1. From In Vitro Fertilization to Family: A Journey with Sher Fertility Solutions (SFS) ; https://sherfertilitysolutions.com/sher-fertility-solutions-ebook.pdf

 

  1. Recurrent Pregnancy Loss and Unexplained IVF Failure: The Immunologic Link ;https://drive.google.com/file/d/1iYKz-EkAjMqwMa1ZcufIloRdxnAfDH8L/view

 

I invite you to visit my very recently launched “Podcast”,  “HAVE A BABY” on RUMBLE;   https://rumble.com/c/c-3304480

If you are interested in having an online consultation with me, please contact my assistant, Patti Converse at 702-533-2691 or email her at concierge@sherivf.com\

 

Name: Maria M

He realizado mi primer ciclo para vitrificar ovulos, sin embargo de 5 fólicos que han engordado sólo se ha podido obtener un óvulo maduro.

Tengo 37 años y AMH de 0,33. En la ecografía se observaban 10 folículos.

El tratamiento del primer ciclo ha sido:
– Inicio del tratamiento: 3/4 día de regla ya que hay problemas para detectarlo porque inicio con manchado y no con sangre. El dia anterior inicio tratamiento: estradiol a 75 y 7 folículos antrales.l (ya menos que las otras ecografías semanas antes que eran 10).
– 1 y 2 dia de tratamiento se ha suministrado meriofert 150 UI, mientras que del 3 al 9 se ha puesto dosis de 300 UI.
– Día 6 de tratamiento se ven dos folículos muy evolucionados (17-15, mientras el resto a 6-6-8)
– 5 al 9 dia tratamiento se ha suministrado orgalutran 0,25
– el 10 dia a las 21:30 se ha suministrado decapeptyl 0,2 y ovitrelle 250 “dual trigger”. Tamaño: 23-19-15-14-13
– del 1 al 9 del tratamiento se ha administado cleoxane 40mg por riesgo trombosis
– el día 11 del tratamiento 9:40 se realiza punción y sólo se obtiene uno maduro el que el día 5 medía 8mm y el día 10, 15mm. El resto de folículos el aspirado sale vacío.

Mis dudas son:

(i) ¿ha podido influenciar el día de inicio del tratamiento y que se haya suministado los dos primeros días una dosis menor de meriofert (150 vs los 300 que correspondía)?

(ii) ¿Es necesario suministrar 300 ui o con mi número folículos / reserva con 150 ui no se va a notar diferencia?

(iii) Ahora dicen incluir anticonceptivos previos, según indica pareciera ser adecuado. ¿Cuándo y como?

(iv) ¿A su parecer cómo debería ser el protocolo más adecuado? Me dicen que vuelva a probar ciclo entero pero me preocupa que sólo sea 1 y la carga hormonal (antecedente cáncer de mama).

(v) Así mismo he leído estudios que denotan que si se suministra 36 horas antes de la punción ovitrelle, y luego 12 horas después otra dosis – esto es 24 h antes de la punción – puede mejorar que los folículos si desprendan el óvulo porque puede ser que el aspirado salga vacío a pesar de que haya óvulo adherido entre el cumulus oophorus y la pared folicular pero que el aspirado no sea capaz de absorber y con esta segunda dosis a las 12 horas permite mayor relajación. Otra opción en estudios he leído que es doble aspirado. ¿Tiene usted experiencia o apoya alguna?

(vi) Así mismo puede ser que si están vacíos y se repite la punción al día siguiente puedan desprenderse?

(vii) ¿Hay alguna técnica o medidor que prediga que salgan vacíos? Tenía el estradiol a 900 el día 10.

(viii) el clexane / heparina ha podido afectar?

(ix) ¿Es recomendable tomar complementos coencima Q10 / acido / vitaminas/etc? ¿Recomienda alguna concreta?

Millones de gracias, porque estoy dudando si he escogido la clínica correcta y no se ha seguido el protocolo y debo buscar alternativas o es correcto y debo asumir la realidad?

Saludos

Answer:

Please re-post your question in English!

 

Geoff Sher

___________________________________________________________________________

PLEASE SHARE THIS WITH OTHERS AND HELP SPREAD THE WORD!!

 

Herewith are  online links to 2  E-books recently  co-authored with  my partner at SFS-NY  (Drew Tortoriello MD)……. for your reading pleasure:

  1. From In Vitro Fertilization to Family: A Journey with Sher Fertility Solutions (SFS) ; https://sherfertilitysolutions.com/sher-fertility-solutions-ebook.pdf

 

  1. Recurrent Pregnancy Loss and Unexplained IVF Failure: The Immunologic Link ;https://drive.google.com/file/d/1iYKz-EkAjMqwMa1ZcufIloRdxnAfDH8L/view

 

I invite you to visit my very recently launched “Podcast”,  “HAVE A BABY” on RUMBLE;   https://rumble.com/c/c-3304480

If you are interested in having an online consultation with me, please contact my assistant, Patti Converse at 702-533-2691 or email her at concierge@sherivf.com\

 

Name: Laura S

Primero agradecer su trabajo.

Le escribo para saber según su opinión ¿Cuál es el mejor protocolo de medicamentos para una estimulación ovárica para mi con 37 años y 0,3 de antimulleriana?

Con otras respuestas no consigo entender día a día que debería ser lo propio.

Primer ciclo sólo salió un óvulo maduro de 7 folículos que se veían. ¿Cómo evitar ese efecto?

Muchas gracias!

Answer:

Please post in English!

Understanding the impact of age and ovarian reserve on the success of in vitro fertilization (IVF) is crucial when it comes to reproductive health. This article aims to simplify and clarify these concepts, emphasizing their significance in the selection of ovarian stimulation protocols for IVF. By providing you with this information, we hope to shed light on the importance of considering these factors and making informed decisions regarding fertility treatments.

  1. The Role of Eggs in Chromosomal Integrity: In the process of creating a healthy embryo, it is primarily the egg that determines the chromosomal integrity, which is crucial for the embryo’s competency. A competent egg possesses a normal karyotype, increasing the chances of developing into a healthy baby. It’s important to note that not all eggs are competent, and the incidence of irregular chromosome numbers (aneuploidy) increases with age.
  2. Meiosis and Fertilization: Following the initiation of the LH surge or the hCG trigger shot, the egg undergoes a process called meiosis, halving its chromosomes to 23. During this process, a structure called the polar body is expelled from the egg, while the remaining chromosomes are retained. The mature sperm, also undergoing meiosis, contributes 23 chromosomes. Fertilization occurs when these chromosomes combine, resulting in a euploid embryo with 46 chromosomes. Only euploid embryos are competent and capable of developing into healthy babies.
  3. The Significance of Embryo Ploidy: Embryo ploidy, referring to the numerical chromosomal integrity, is a critical factor in determining embryo competency. Aneuploid embryos, which have an irregular number of chromosomes, are often incompetent and unable to propagate healthy pregnancies. Failed nidation, miscarriages, and chromosomal birth defects can be linked to embryo ploidy issues. Both egg and sperm aneuploidy can contribute, but egg aneuploidy is usually the primary cause.
  4. Embryo Development and Competency: Embryos that develop too slowly or too quickly, have abnormal cell counts, contain debris or fragments, or fail to reach the blastocyst stage are often aneuploid and incompetent. Monitoring these developmental aspects can provide valuable insights into embryo competency.
  5. Diminished Ovarian Reserve (DOR): As women advance in their reproductive age, the number of remaining eggs in the ovaries decreases. Diminished ovarian reserve (DOR) occurs when the egg count falls below a certain threshold, making it more challenging to respond to fertility drugs effectively. This condition is often indicated by specific hormone levels, such as elevated FSH and decreased AMH. DOR can affect women over 40, but it can also occur in younger

 

Why IVF should be regarded as treatment of choice for older women an those who have diminished ovarian reserve ( DOR):

Understanding the following factors will go a long way in helping you to make an informed decision and thereby improve the chances of a successful IVF outcome.

  1. Age and Ovarian Reserve: Chronological age plays a vital role in determining the quality of eggs and embryos. As women age, there is an increased risk of aneuploidy (abnormal chromosome numbers) in eggs and embryos, leading to reduced competency. Additionally, women with declining ovarian reserve (DOR), regardless of their age, are more likely to have aneuploid eggs/embryos. Therefore, it is crucial to address age-related factors and ovarian reserve to enhance IVF success.
  2. Excessive Luteinizing Hormone (LH) and Testosterone Effects: In women with DOR, their ovaries and developing eggs are susceptible to the adverse effects of excessive LH, which stimulates the overproduction of male hormones like testosterone. While some testosterone promotes healthy follicle growth and egg development, an excess of testosterone has a negative impact. Therefore, in older women or those with DOR, ovarian stimulation protocols that down-regulate LH activity before starting gonadotropins are necessary to improve egg/embryo quality and IVF outcomes.
  3. Individualized Ovarian Stimulation Protocols: Although age is a significant factor in aneuploidy, it is possible to prevent further decline in egg/embryo competency by tailoring ovarian stimulation protocols. Here are my preferred protocols for women with relatively normal ovarian reserve:
  1. Conventional Long Pituitary Down Regulation Protocol:
  • Begin birth control pills (BCP) early in the cycle for at least 10 days.
  • Three days before stopping BCP, overlap with an agonist like Lupron for three days.
  • Continue daily Lupron until menstruation begins.
  • Conduct ultrasound and blood estradiol measurements to assess ovarian status.
  • Administer FSH-dominant gonadotropin along with Menopur for stimulation.
  • Monitor follicle development through ultrasound and blood estradiol measurements.
  • Trigger egg maturation using hCG injection, followed by egg retrieval.
  1. Agonist/Antagonist Conversion Protocol (A/ACP):
  • Similar to the conventional long down regulation protocol but replace the agonist with a GnRH antagonist from the onset of post-BCP menstruation until the trigger day.
  • Consider adding supplementary human growth hormone (HGH) for women with DOR.
  • Consider using “priming” with estrogen prior to gonadotropin administration
  1. Protocols to Avoid for Older Women or Those with DOR: Certain ovarian stimulation protocols may not be suitable for older women or those with declining ovarian reserve:
  • Microdose agonist “flare” protocols
  • High dosages of LH-containing fertility drugs such as Menopur
  • Testosterone-based supplementation
  • DHEA supplementation
  • Clomiphene citrate or Letrozole
  • Low-dosage hCG triggering or agonist triggering for women with DOR

 

 

Preimplantation Genetic Screening/Testing(PGS/T): PGS/T is a valuable tool for identifying chromosomal abnormalities in eggs and embryos. By selecting the most competent (euploid) embryos, PGS/T significantly improves the success of IVF, especially in older women or those with DOR.

Understanding the impact of advancing age and declining ovarian reserve on IVF outcomes is essential when making decisions about fertility treatments. Age-related factors can affect egg quality and increase the likelihood of aneuploid embryos with resultant IVF failure. Diminished ovarian reserve (DOR) further complicates the process. By considering these factors, you can make informed choices and work closely with fertility specialists to optimize your chances of success. Remember, knowledge is power, and being aware of these aspects empowers you to take control of your reproductive journey.

 _______________________________________________________________________________

PLEASE SHARE THIS WITH OTHERS AND HELP SPREAD THE WORD!!

 

Herewith are  online links to 2  E-books recently  co-authored with  my partner at SFS-NY  (Drew Tortoriello MD)……. for your reading pleasure:

  1. From In Vitro Fertilization to Family: A Journey with Sher Fertility Solutions (SFS) ; https://sherfertilitysolutions.com/sher-fertility-solutions-ebook.pdf

 

  1. Recurrent Pregnancy Loss and Unexplained IVF Failure: The Immunologic Link ;https://drive.google.com/file/d/1iYKz-EkAjMqwMa1ZcufIloRdxnAfDH8L/view

 

I invite you to visit my very recently launched “Podcast”,  “HAVE A BABY” on RUMBLE;   https://rumble.com/c/c-3304480

If you are interested in having an online consultation with me, please contact my assistant, Patti Converse at 702-533-2691 or email her at concierge@sherivf.com\

 

 

Name: Elizabeth M

Hello Dr Sher

I’m 42 years old with an AMH of 4.4 pmol/L.

How many units of Lupron/Puregon/Menopur /HGH would you recommend for me in this type of protocol?

Answer:

WE should talk! There are too many other factors to be considered!

ADDRESSING ADVANCING AGE AND DIMINISHING OVARIAN RESERVE (DOR) IN IVF

Geoffrey Sher MD

Understanding the impact of age and ovarian reserve on the success of in vitro fertilization (IVF) is crucial when it comes to reproductive health. This article aims to simplify and clarify these concepts, emphasizing their significance in the selection of ovarian stimulation protocols for IVF. By providing you with this information, we hope to shed light on the importance of considering these factors and making informed decisions regarding fertility treatments.

  1. The Role of Eggs in Chromosomal Integrity: In the process of creating a healthy embryo, it is primarily the egg that determines the chromosomal integrity, which is crucial for the embryo’s competency. A competent egg possesses a normal karyotype, increasing the chances of developing into a healthy baby. It’s important to note that not all eggs are competent, and the incidence of irregular chromosome numbers (aneuploidy) increases with age.
  2. Meiosis and Fertilization: Following the initiation of the LH surge or the hCG trigger shot, the egg undergoes a process called meiosis, halving its chromosomes to 23. During this process, a structure called the polar body is expelled from the egg, while the remaining chromosomes are retained. The mature sperm, also undergoing meiosis, contributes 23 chromosomes. Fertilization occurs when these chromosomes combine, resulting in a euploid embryo with 46 chromosomes. Only euploid embryos are competent and capable of developing into healthy babies.
  3. The Significance of Embryo Ploidy: Embryo ploidy, referring to the numerical chromosomal integrity, is a critical factor in determining embryo competency. Aneuploid embryos, which have an irregular number of chromosomes, are often incompetent and unable to propagate healthy pregnancies. Failed nidation, miscarriages, and chromosomal birth defects can be linked to embryo ploidy issues. Both egg and sperm aneuploidy can contribute, but egg aneuploidy is usually the primary cause.
  4. Embryo Development and Competency: Embryos that develop too slowly or too quickly, have abnormal cell counts, contain debris or fragments, or fail to reach the blastocyst stage are often aneuploid and incompetent. Monitoring these developmental aspects can provide valuable insights into embryo competency.
  5. Diminished Ovarian Reserve (DOR): As women advance in their reproductive age, the number of remaining eggs in the ovaries decreases. Diminished ovarian reserve (DOR) occurs when the egg count falls below a certain threshold, making it more challenging to respond to fertility drugs effectively. This condition is often indicated by specific hormone levels, such as elevated FSH and decreased AMH. DOR can affect women over 40, but it can also occur in younger

 

Why IVF should be regarded as treatment of choice for older women an those who have diminished ovarian reserve ( DOR):

Understanding the following factors will go a long way in helping you to make an informed decision and thereby improve the chances of a successful IVF outcome.

  1. Age and Ovarian Reserve: Chronological age plays a vital role in determining the quality of eggs and embryos. As women age, there is an increased risk of aneuploidy (abnormal chromosome numbers) in eggs and embryos, leading to reduced competency. Additionally, women with declining ovarian reserve (DOR), regardless of their age, are more likely to have aneuploid eggs/embryos. Therefore, it is crucial to address age-related factors and ovarian reserve to enhance IVF success.
  2. Excessive Luteinizing Hormone (LH) and Testosterone Effects: In women with DOR, their ovaries and developing eggs are susceptible to the adverse effects of excessive LH, which stimulates the overproduction of male hormones like testosterone. While some testosterone promotes healthy follicle growth and egg development, an excess of testosterone has a negative impact. Therefore, in older women or those with DOR, ovarian stimulation protocols that down-regulate LH activity before starting gonadotropins are necessary to improve egg/embryo quality and IVF outcomes.
  3. Individualized Ovarian Stimulation Protocols: Although age is a significant factor in aneuploidy, it is possible to prevent further decline in egg/embryo competency by tailoring ovarian stimulation protocols. Here are my preferred protocols for women with relatively normal ovarian reserve:
  1. Conventional Long Pituitary Down Regulation Protocol:
  • Begin birth control pills (BCP) early in the cycle for at least 10 days.
  • Three days before stopping BCP, overlap with an agonist like Lupron for three days.
  • Continue daily Lupron until menstruation begins.
  • Conduct ultrasound and blood estradiol measurements to assess ovarian status.
  • Administer FSH-dominant gonadotropin along with Menopur for stimulation.
  • Monitor follicle development through ultrasound and blood estradiol measurements.
  • Trigger egg maturation using hCG injection, followed by egg retrieval.
  1. Agonist/Antagonist Conversion Protocol (A/ACP):
  • Similar to the conventional long down regulation protocol but replace the agonist with a GnRH antagonist from the onset of post-BCP menstruation until the trigger day.
  • Consider adding supplementary human growth hormone (HGH) for women with DOR.
  • Consider using “priming” with estrogen prior to gonadotropin administration
  1. Protocols to Avoid for Older Women or Those with DOR: Certain ovarian stimulation protocols may not be suitable for older women or those with declining ovarian reserve:
  • Microdose agonist “flare” protocols
  • High dosages of LH-containing fertility drugs such as Menopur
  • Testosterone-based supplementation
  • DHEA supplementation
  • Clomiphene citrate or Letrozole
  • Low-dosage hCG triggering or agonist triggering for women with DOR

 

 

Preimplantation Genetic Screening/Testing(PGS/T): PGS/T is a valuable tool for identifying chromosomal abnormalities in eggs and embryos. By selecting the most competent (euploid) embryos, PGS/T significantly improves the success of IVF, especially in older women or those with DOR.

Understanding the impact of advancing age and declining ovarian reserve on IVF outcomes is essential when making decisions about fertility treatments. Age-related factors can affect egg quality and increase the likelihood of aneuploid embryos with resultant IVF failure. Diminished ovarian reserve (DOR) further complicates the process. By considering these factors, you can make informed choices and work closely with fertility specialists to optimize your chances of success. Remember, knowledge is power, and being aware of these aspects empowers you to take control of your reproductive journey.

_______________________________________________________________________________________________________________________________________

Herewith are  online links to 2  E-books recently  co-authored with  my partner at SFS-NY  (Drew Tortoriello MD)……. for your reading pleasure:

  1. From In Vitro Fertilization to Family: A Journey with Sher Fertility Solutions (SFS) ; https://sherfertilitysolutions.com/sher-fertility-solutions-ebook.pdf

 

  1. Recurrent Pregnancy Loss and Unexplained IVF Failure: The Immunologic Link ;https://drive.google.com/file/d/1iYKz-EkAjMqwMa1ZcufIloRdxnAfDH8L/view

 

I invite you to visit my very recently launched “Podcast”,  “HAVE A BABY” on RUMBLE;   https://rumble.com/c/c-3304480

If you are interested in having an online consultation with me, please contact my assistant, Patti Converse at 702-533-2691 or email her at concierge@sherivf.com\

 

 

 

 

Name: Danille M

Hello, I am trying to conceive and have been told the issue is I have a 5cm endometrioma. I am hoping to have this aspirated before starting any treatments. I do not want surgery to remove it as they want to take my ovary. Please let me know and if you do this or know someone who does I would love to make an appointment.

Answer:

Advanced endometriosis is often associated with ovarian endometriotic cysts, known as endometriomas. In my opinion, such cysts, if numerous or sizable (>1cm) can and often do activate the surrounding ovarian connective tissue to locally produce excessive amounts of male hormones such as testosterone. This effect can often not be detected in the blood but nevertheless occurs in the affected ovary where, upon entering the follicles, it can i(n my opinion) compromise egg development which in turn often can result in abnormal egg development during ovarian stimulation. This can in my opinion, lead to increased numerical chromosomal abnormalities (aneuploidy), reducing egg/embryo competency”. Thus it is my opinion that any ovarian endometriomas larger than 1cm in diameter, should be removed eliminated before embarking on IVF.

Aside from compromising egg quality in the involved ovary, endometriomas can also cause severe and intractable pelvic pain, heavy menstruation (menometrorrhagia) and painful intercourse. They can also rupture leading to dissemination throughout the abdominal-pelvic cavity.  Accordingly, they are best addressed sooner rather than later.

SurgicalTreatment: Conventional treatment of endometriomas involves complete surgical removal, usually conducted laparoscopically. Unfortunately, with surgery normal ovarian tissue can inadvertently be removed/damaged and result in scarring that that can compromise subsequent egg development in the affected ovary. Since many women who have endometriomas have already undergone previous surgery (ies) for endometriosis. Both the surgery (ies) and the advanced endometriosis will often have resulted in significant scarring and adhesion formation that can compromise subsequent visualization of, and access to anatomic structures during surgery, thereby increasing the risk of surgical complications. As such, many women with recurrent ovarian endometriomas are uncomfortable with the prospect of repeat surgery and its avoidance is often a factor in their decision to proceed with IVF.

Aspiration of sizeable endometriomas is unfortunately not a solution. The cyst will return! Removal surgically or laser ablation of the inner lining of the endometrioma is needed. And in the hands of an expert, the ovary need not be sacrificed. Failure to address removal or ablation will usually ensure that the vast majority of eggs produced by the affected ovary will be ‘incompetent”.

Geoff Sher

______________________________________________________________

 

Herewith are  online links to 2  E-books recently  co-authored with  my partner at SFS-NY  (Drew Tortoriello MD)……. for your reading pleasure:

  1. From In Vitro Fertilization to Family: A Journey with Sher Fertility Solutions (SFS) ; https://sherfertilitysolutions.com/sher-fertility-solutions-ebook.pdf

 

  1. Recurrent Pregnancy Loss and Unexplained IVF Failure: The Immunologic Link ;https://drive.google.com/file/d/1iYKz-EkAjMqwMa1ZcufIloRdxnAfDH8L/view

 

I invite you to visit my very recently launched “Podcast”,  “HAVE A BABY” on RUMBLE;   https://rumble.com/c/c-3304480

If you are interested in having an online consultation with me, please contact my assistant, Patti Converse at 702-533-2691 or email her at concierge@sherivf.com\

Name: Kimberly N

Hi Dr. Sher,

I’m here because, like many women who come to you, I have had multiple failed FETs. I did IVF back in 2017 due to male Factor infertility, azoospermia. I was 30 years old and my embryos were all highly graded.

My first transfer was successful, I have an almost 6 year old son. I had transferred to embryos and one survived, my son who was born at 30 weeks premature.

About a year and a half ago, I transferred another highly graded embryo, that was healthy and all scans, until I lost him at 13 weeks gestation. After some testing I found out that I have APS. Due to anemia I had to get better before I could do my next transfer. I did my next transfer in September and another one in October both of which failed to implant. On my second transfer of this year, my RE added Lovenox, aspirin, methylprednisone, claritin, and antibiotics which I took prior to the transfer that still failed.

I have one more embryo left. Can you help me find the right protocol so that this last embryo sticks?

Answer:

This very much sounds like an implantation dysfunction

Implantation dysfunction is often overlooked as a significant reason for IVF failure. This is especially true when IVF failure is unexplained, or when there are recurring pregnancy losses or underlying issues with the uterus, such as endo-uterine surface lesions, thin uterine lining (endometrium), or immunological factors.

IVF success rates have been improving in the past decade. Currently, in the United States, the average live birth rate per embryo transfer for women under 40 years old using their own eggs is about 2:5 per woman undergoing embryo transfer. However, there is a wide range of success rates among different IVF programs, varying from 20% to almost 50%. Based on these statistics, most women in the United States need to undergo two or more IVF-embryo transfer attempts to have a baby. Many IVF practitioners in the United States attribute the differences in success rates to variations in expertise among embryology laboratories, but this is not entirely accurate. Other factors, such as differences in patient selection, the failure to develop personalized protocols for ovarian stimulation, and the neglect of infectious, anatomical, and immunological factors that affect embryo implantation, are equally important.

Approximately 80% of IVF failures occur due to “embryo incompetency,” mainly caused by ( irregularities in chromosome number (aneuploidy), which is often related to the advancing age of the woman, diminished ovarian reserve ( DOR) but can also be influenced by the ovarian stimulation protocol chosen, and sperm dysfunction (male infertility). However, in around 20% of cases with dysfunction, failure is caused by problems with embryo implantation.

This section will focus on embryo implantation dysfunction and IVF failure which in the vast majority of cases is caused by:

  1. 1. Anatomical irregularities of the inner uterine surface:
  2. a) Surface lesions such as polyps/fibroids/ scar tissue
  3. b)endometrial thickness
  4.  
  5. 2. Immunologic Implantation Dysfunction ( IID)lesions
  6. a)Autoimmune IID
  7. b) Alloimmune IID

  1. ANATOMICAL IMPLANTATION DYSFUNCTION
  2. a) Surface lesions such as polyps/fibroids/ scar tissue

When there are problems with the structure of the uterus, it can lead to difficulties in getting pregnant. While uterine fibroids usually don’t cause infertility, they can affect fertility when they distort the uterine cavity or protrude through the lining. Even small fibroids located just beneath the lining and protruding into the cavity can decrease the chances of the embryo attaching. Multiple fibroids within the uterine wall that encroach upon the cavity can disrupt blood flow, impair estrogen delivery, and prevent proper thickening of the lining. These issues can be identified through ultrasound during the menstrual cycle’s proliferative phase. Any lesion on the uterine surface, such as submucous fibroids, adhesions, endometrial polyps, or placental polyps, can interfere with implantation by causing a local inflammatory response similar to the effect of an intrauterine contraceptive device (IUD).

Clearly, even small uterine lesions can have a negative impact on implantation. Considering the high costs and emotional toll associated with in vitro fertilization (IVF) and related procedures, it is reasonable to perform diagnostic tests like hysterosalpingography (HSG), fluid ultrasound examination (hysterosonogram), or hysteroscopy before starting IVF. Uterine lesions that can affect implantation often require surgical intervention. In most cases, procedures like dilatation and curettage (D&C) or hysteroscopic resection are sufficient. Rarely a laparotomy may be needed. Such interventions often lead to an improvement in the response of the uterine lining.

Hysterosonogram( HSN/saline ultrasound) is a procedure where a sterile saline solution is injected into the uterus through the cervix using a catheter. Vaginal ultrasound is then used to examine the fluid-filled cavity for any irregularities that might indicate surface lesions like polyps, fibroid tumors, scarring, or a septum. When performed by an expert, HSN is highly effective in detecting even the smallest lesions and can supplant hysteroscopy in certain cases. HSN is less expensive, less invasive/traumatic, and equally effective as hysteroscopy. The only drawback is that if a lesion is found, hysteroscopy may still be needed for treatment.

Hysteroscopy is a diagnostic procedure performed in an office setting with minimal discomfort to the patient. It involves inserting a thin, lighted instrument called a hysteroscope through the vagina and cervix into the uterus to examine the uterine cavity. Normal saline is used to distend the uterus during the procedure. Like HSN, hysteroscopy allows for direct visualization of the inside of the uterus to identify defects that could interfere with implantation. We have observed that around one in eight IVF candidates have lesions that need attention before undergoing IVF to optimize the chances of success. I strongly recommend that all patients undergo therapeutic surgery, usually hysteroscopy, to correct any identified issues before proceeding with IVF. Depending on the severity and nature of the problem, hysteroscopy may require general anesthesia and should be performed in a surgical facility equipped for laparotomy if necessary.

  1. b) Thickness of the uterine lining (endometrium)

As far back as In 1989, I and my team made an important discovery about using ultrasound to assess the thickness of the endometrium during the late proliferative phase of both “ natural” and hormone-stimulated cycles. The assessment helped predict the chances of conception. We found that an ideal thickness of over 9mm at the time of ovulation , egg retrieval or with the commencement of progesterone therapy in embryo recipient cycles ( e.g., IVF with egg donation, gestational, surrogacy and embryo adoption) was associated with optimal implantation rates, while an endometrial thickness of less than 8 mm was associated with failure to implant or early pregnancy loss in the vast majority of cases. An endometrium measuring <8mm was almost invariably associated with failure to implant or early pregnancy loss in the while an endometrium measuring 8 to 9 mm was regarded as being intermediate, and while pregnancies did occur in this range, the rates were only slightly lower than with an optimal lining of 9 mm

A “poor” uterine lining typically occurs when the innermost layer of the endometrium (basal or germinal endometrium) is unable to respond to estrogen by developing a thick enough outer “functional” layer to support successful embryo implantation and placental development. The “functional” layer, which accounts for two-thirds of the total endometrial thickness, is shed during menstruation if pregnancy does not occur.

The main causes of a poor uterine lining are:

  1. Damage to the basal endometrium due to:
    • Inflammation of the endometrium (endometritis), often resulting from retained products of conception after abortion, miscarriage, or childbirth.
    • Surgical trauma caused by aggressive dilatation and curettage (D&C).
  1. Insensitivity of the basal endometrium to estrogen due to:
    • Prolonged (back to back) use of clomiphene citrate for ovarian stimulation or…
    • Prenatal exposure to diethylstilbestrol (DES), a drug given to prevent miscarriage in the 1960s.
  1. Overexposure of the uterine lining to male hormones produced by the ovaries or administered during ovarian stimulation (primarily testosterone):
    • Older women, women with DOR (poor responders), and women with polycystic ovarian syndrome (PCOS) often have increased biological activity of luteinizing hormone (LH), leading to testosterone overproduction by the ovarian connective tissue (stroma/theca). This effect can be further amplified when certain ovarian stimulation protocols were high doses of menotropins ( e.g., Menopur) are used.
  1. Reduced blood flow to the basal endometrium caused by:
    • Multiple uterine fibroids, especially if they are located beneath the endometrium (submucosal).
    • Uterine adenomyosis, which involves extensive abnormal invasion of endometrial glands into the uterine muscle.

In 1996 I introduced the Vaginal administration of Sildenafil (Viagra) to improve endometrial thickening. The selective administration of Sildenafil has shown great promise in improving uterine blood flow and increasing endometrial thickening in cases of thin endometrial linings. When administered vaginally, it is quickly absorbed and reaches high concentrations in the uterine blood system, diluting as it enters the systemic circulation. This method has been found to have minimal systemic side effects. However, it is important to note that Viagra may not be effective in all cases, as some cases of thin uterine linings may involve permanent damage to the basal endometrium, rendering it unresponsive to estrogen.

Severe endometrial damage leading to poor responsiveness to estrogen can occur in various situations. These include post-pregnancy endometritis (inflammation after childbirth), chronic granulomatous inflammation caused by uterine tuberculosis (rare in the United States), and significant surgical injury to the basal endometrium (which can happen after aggressive D&C procedures).

 

  1. IMMUNOLOGIC IMPLANTATION DYSFUNCTION (IID)

There is a growing recognition that problems with the immune function in the uterus can lead to embryo implantation dysfunction. The failure of proper immunologic interaction during implantation has been implicated as a cause of recurrent miscarriage, late pregnancy fetal loss, IVF failure, and infertility. Some immunologic factors that may contribute to these issues include antiphospholipid antibodies (APA), antithyroid antibodies (ATA) , and activated natural killer cells (NKa).

  • Activated natural Killer Cells (NKa):

During ovulation and early pregnancy, the uterine lining is frequented by NK cells and T-cells, which together make up more than 80% of the immune cells in the uterine lining. These cells travel from the bone marrow to the endometrium where they proliferate under hormonal regulation. When exposed to progesterone, they produce TH-1 and TH-2 cytokines. TH-2 cytokines help the trophoblast (embryo’s “root system”) to penetrate the uterine lining, while TH-1 cytokines induce apoptosis (cell suicide), limiting placental development to the inner part of the uterus. The balance between TH1 and TH-2 cytokines is crucial for optimal placental development. NK cells and T-cells contribute to cytokine production. Excessive TH-1 cytokine production is harmful to the trophoblast and endometrial cells, leading to programmed cell death and ultimately to implantation failure. Functional NK cells reach their highest concentration in the endometrium around 6-7days after ovulation or exposure to progesterone, which coincides with the time of embryo implantation. It’s important to note that measuring the concentration of blood NK cells doesn’t reflect NK cell activation (NKa). The activation of NK cells is what matters. In certain conditions like endometriosis, the blood concentration of NK cells may be below normal, but NK cell activation is significantly increased.

There are several laboratory methods to assess NK cell activation (cytotoxicity), including immunohistochemical assessment of uterine NK cells and measuring TH-1 cytokines in the uterus or blood. However, the K-562 target cell blood test remains the gold standard. In this test, NK cells isolated from a woman’s blood are incubated with specific “target cells,” and the percentage of killed target cells is quantified. More than 12% killing indicates a level of NK cell activation that usually requires treatment. Currently, there are only a few Reproductive Immunology Reference Laboratories in the USA capable of reliably performing the K-562 target cell test.

There is a common misconception that adding IL (intralipid) or Intravenous gammaglobulin (IVIg) to NK cells can immediately downregulate NK cell activity. However, neither IL and IVIg cannot significantly suppress already activated NK cells. They are believed to work by regulating NK cell progenitors, which then produce downregulated NK cells. To assess the therapeutic effect, IL/IVIg infusion should be done about 14 days before embryos are transferred to the uterus to ensure a sufficient number of normal functional NK cells are present at the implantation site during embryo transfer. Failure to recognize this reality has led to the erroneous demand from IVF doctors for Reproductive Immunology Reference Laboratories to report on NK cell activity before and immediately after exposure to IVIg or IL at different concentrations. However, since already activated NK cells cannot be deactivated in the laboratory, assessing NKa suppression in this way has little clinical benefit. Even if blood is drawn 10-14 days after IL/IVIg treatment, it would take another 10-14 days to receive the results, which would be too late to be practically advantageous.

  • Antiphospholipid Antibodies:

Many women who struggle with IVF failure or recurrent pregnancy loss, as well as those with a personal or family history of autoimmune diseases like lupus erythematosus, rheumatoid arthritis, scleroderma, and dermatomyositis, often test positive for antiphospholipid antibodies (APAs). Over 30 years ago, I proposed a treatment for women with positive APA tests. This involved using a low dose of heparin to improve the success of IVF implantation and increase birth rates. Research indicated that heparin could prevent APAs from affecting the embryo’s “root system” ( the trophoblast), thus enhancing implantation. We later discovered that this therapy only benefits women whose APAs target specific phospholipids (phosphatidylethanolamine and phosphatidylserine). Nowadays, longer-acting low molecular weight heparinoids like Lovenox and Clexane have replaced heparin.

  • Antithyroid Antibodies ( thyroid peroxidase  -TPO and antithyroglobulin antibodies (TGa)

Between 2% and 5% of women of the childbearing age have reduced thyroid hormone activity (hypothyroidism). Women with hypothyroidism often manifest with reproductive failure i.e., infertility, unexplained (often repeated) IVF failure, or recurrent pregnancy loss (RPL). The condition is 5-10 times more common in women than in men. In most cases hypothyroidism is caused by damage to the thyroid gland resulting from thyroid autoimmunity (Hashimoto’s disease) caused by damage done to the thyroid gland by antithyroglobulin and antimicrosomal auto-antibodies. The increased prevalence of hypothyroidism and thyroid autoimmunity (TAI) in women is likely the result of a combination of genetic factors, estrogen-related effects, and chromosome X abnormalities. This having been said, there is significantly increased incidence of thyroid antibodies in non-pregnant women with a history of infertility and recurrent pregnancy loss and thyroid antibodies can be present asymptomatically in women without them manifesting with overt clinical or endocrinologic evidence of thyroid disease. In addition, these antibodies may persist in women who have suffered from hyper- or hypothyroidism even after normalization of their thyroid function by appropriate pharmacological treatment. The manifestations of reproductive dysfunction thus seem to be linked more to the presence of thyroid autoimmunity (TAI) than to clinical existence of hypothyroidism and treatment of the latter does not routinely result in a subsequent improvement in reproductive performance. It follows that if antithyroid autoantibodies are associated with reproductive dysfunction they may serve as useful markers for predicting poor outcome in patients undergoing assisted reproductive technologies. Some years back, I reported on the fact that 47% of women who harbor thyroid autoantibodies, regardless of the absence or presence of clinical hypothyroidism, have activated uterine natural killer cells (NKa) cells and cytotoxic lymphocytes (CTL) and that such women often present with reproductive dysfunction. We demonstrated that appropriate immunotherapy with IVIG or intralipid (IL) and steroids subsequently often results in a significant improvement in reproductive performance in such cases.


Almost 50% of women with antithyroid antibodies do not have activated cytotoxic T lymphocytes (CTL) or natural killer cells (NK cells). This suggests that the antibodies themselves may not be the direct cause of reproductive dysfunction. Instead, the activation of CTL and NK cells, which occurs in about half of the cases with thyroid autoimmunity (TAI), is likely an accompanying phenomenon that damages the early “root system” (trophoblast) of the embryo during implantation.

Treating women who have both antithyroid antibodies and activated NK cells/CTL with intralipid (IL) and steroids improves their chances of successful reproduction. However, women with antithyroid antibodies who do not have activated NK cells/CTL do not require this treatment.

  • Treatment Options for IID:
  1. Intralipid (IL) Therapy: IL is a mixture of soybean lipid droplets in water, primarily used for providing nutrition. When administered intravenously, IL supplies essential fatty acids that can activate certain receptors in NK cells, reducing their cytotoxic activity and enhancing implantation. IL, combined with corticosteroids, suppresses the overproduction of pro-inflammatory cytokines by NK cells, improving reproductive outcomes. IL is cost-effective and has fewer side effects compared to other treatments like IVIg.
  2. Intravenous immunoglobulin-G (IVIg) Therapy:In the past, IVIg was used to down-regulate activated NK cells. However, concerns about viral infections and the high cost led to a decline in its use. IVIg can be effective, but IL has become a more favorable and affordable alternative.
  3. Corticosteroid Therapy: Corticosteroids, such as prednisone and dexamethasone, are commonly used in IVF treatment. They have an immunomodulatory effect and reduce TH-1 cytokine production by CTL. When combined with IL or IVIg, corticosteroids enhance the implantation process. Treatment typically starts 10-14 days before embryo transfer and continues until the 10th week of pregnancy.
  4. Heparinoid Therapy: Low molecular weight heparin (Clexane, Lovenox)can improve IVF success rates in women with antiphospholipid antibodies (APAs) and may prevent pregnancy loss in certain thrombophilias when used during treatment. It is administered subcutaneously once daily from the start of ovarian stimulation.
  5. TH-1 Cytokine Blockers (Enbrel, Humira):TH-1 cytokine blockers have limited effectiveness in the IVF setting and, in my opinion, no compelling evidence supports their use. They may have a role in treating threatened miscarriage caused by CTL/NK cell activation, but not for IVF treatment. TH-1 cytokines are needed for cellular response, during the early phase of implantation, so completely blocking them could hinder normal implantation.
  6. Baby Aspirin and IVF:Baby aspirin doesn’t offer much value in treating implantation dysfunction (IID) and may even reduce the chance of success. This is because aspirin thins the blood and increases the risk of bleeding, which can complicate procedures like egg retrieval or embryo transfer during IVF, potentially compromising its success.
  7. Leukocyte Immunization Therapy (LIT):LIT involves injecting the male partner’s lymphocytes into the mother to improve the recognition of the embryo as “self” and prevent rejection. LIT can up-regulate Treg cells and down-regulate NK cell activation, improving the balance of TH-1 and TH-2 cells in the uterus. However, the same benefits can be achieved through IL (Intralipid) therapy combined with corticosteroids. IL is more cost-effective, and the use of LIT is prohibited by law in the USA.

Types of Immunologic Implantation Dysfunction (IID) and NK Cell Activation:

  1. 1.Autoimmune Implantation Dysfunction: Women with a personal or family history of autoimmune conditions like Rheumatoid arthritis, Lupus Erythematosus, thyroid autoimmune disease (Hashimoto’s disease and thyrotoxicosis), and endometriosis (in about one-third of cases) may experience autoimmune IID. However, autoimmune IID can also occur without any personal or family history of autoimmune diseases. Treatment for NK cell activation in IVF cases complicated by autoimmune IID involves a combination of daily oral dexamethasone from the start of ovarian stimulation until the 10th week of pregnancy, along with 20% intralipid (IL) infusion 10 days to 2 weeks before embryo transfer. With this treatment, the chance of a viable pregnancy occurring within two completed embryo transfer  attempts is approximately 70% for women <40 years old who have  normal ovarian reserve.
  2. Alloimmune Implantation Dysfunction:NK cell activation occurs when the uterus is exposed to an embryo that shares certain genotypic (HLA/DQ alpha) similarities with the embryo recipient.
      • Partial DQ alpha/HLA genetic matching: Couples who share only one DQ alpha/HLA gene are considered to have a “partial match.” If NK cell activation is also present, this partial match puts the couple at a disadvantage for IVF success. However, it’s important to note that DQ alpha/HLA matching, whether partial or total, does not cause IID without associated NK cell activation. Treatment for partial DQ alpha/HLA match with NK cell activation involves IL infusion and oral prednisone as adjunct therapy. IL infusion is repeated every 2-4 weeks after pregnancy is confirmed and continued until the 24th week of gestation. In these cases, only one embryo is transferred at a time to minimize the risk of NK cell activation.
      • Total (Complete) Alloimmune Genetic Matching: A total alloimmune match occurs when the husband’s DQ alpha genotype matches both that of the partner. Although rare, this total match along with NK cell activation significantly reduces the chance of a viable pregnancy resulting in a live birth at term. In some cases, the use of a gestational surrogate may be necessary.

It should be emphasized that poor embryo quality is not always the main cause of reproductive dysfunction and that the complex interaction between embryonic cells and the lining of the uterus  plays a critical role in successful implantation. Women with personal or family histories of autoimmune disease or endometriosis and those with unexplained (often repeated) IVF failure or recurrent pregnancy loss, often have immunologic implantation dysfunction (IID as the underlying cause . For such women, it is important to understand how IID leads to reproductive failure and how selective treatment options such as intralipid (IL), corticosteroid and heparinoid therapy, can dramatically  improve reproductive outcomes. Finally, there is real hope that proper identification and management of IID can  significantly improve the chance of successful reproduction and ultimately contribute to better quality of life after birth.

 

______________________________________________________________________

Herewith are  online links to 2  E-books recently  co-authored with  my partner at SFS-NY  (Drew Tortoriello MD)……. for your reading pleasure:

  1. From In Vitro Fertilization to Family: A Journey with Sher Fertility Solutions (SFS) ; https://sherfertilitysolutions.com/sher-fertility-solutions-ebook.pdf

 

  1. Recurrent Pregnancy Loss and Unexplained IVF Failure: The Immunologic Link ;https://drive.google.com/file/d/1iYKz-EkAjMqwMa1ZcufIloRdxnAfDH8L/view

 

I invite you to visit my very recently launched “Podcast”,  “HAVE A BABY” on RUMBLE;   https://rumble.com/c/c-3304480

If you are interested in having an online consultation with me, please contact my assistant, Patti Converse at 702-533-2691 or email her at concierge@sherivf.com\

 

Name: Lou M

Hi Dr Sher

I’m 43 year old woman with PCOS and Type 2 Diabetes and have endometrial hyperplasia atypia. My doctor initially recommend a hysterectomy but I would like to have a family. I realise I will likely require IVF once the hyperplasia is under control due to my age and medical history.

I have undergone multiple hysteroscopy procedures and had the Mirena coil fitted anumber of times. At first the thickness of the lining decreased but more recently that hasn’t happened. In August 2023 hysteroscopy showed that the thickness had increased and i had the previous coil replaced with another. I was prescribed Provera 200mg three times daily to help this issue.

I am having a check up very soon after being on this medication for 10 weeks, and my question to you Dr Sher is, would this be the same way you would approach the issue of a patient with endometrial hyperplasia atypia who would like to have a family? When would I be clear to go forward with IVF? One clinic has recommended egg/embryo banking sooner rather than later.

I have been unable to find a lot of helpful information and want to be sure there’s nothing further I should try or questions to ask my doctor. Any advice would be appreciated.

Thanks
L

Answer:

The commonest form of EH in women with PCOS is cystic glandular hyperplasia and this is not commonly associated with atypia and is not premalignant. It can be suppressed with progestational therapy and alone does not preclude pregnancy. Another form of hyperplasia that sometimes occurs with PCOSi s adenomatous hyperplasia and this IS ASSOCIATED with atypia and can be pemalignant. In my opinion, if what you have is the former, then you should be able to proceed but if it is the latter, I would have to agree with your doctor.

This having been said, separately,. you should also consider your age and its impact on IVF success.

Overcoming Age Barriers in IVF and Embracing New Pathways to Parenthood: Unlocking the Possibilities.

Geoffrey Sher MD

 

Age should never be a barrier to hope and fulfillment when it comes to IVF. Many women in their early to mid-40s are successfully having IVF babies using their own eggs, especially if they have a good number of eggs left in their ovaries. However, for women with diminished ovarian reserve (DOR) or those over the age of 44, where the chances of success with their own eggs are low, IVF with egg donation can be a highly successful and safe option. Let’s explore why age affects IVF outcomes and discover the possibilities that lie ahead.

The egg plays a crucial role in determining the quality of the embryo, with a “competent” egg having the best chance of developing into a healthy baby. As women age, the chances of having eggs with an irregular number of chromosomes (aneuploid) increase significantly. Fertilizing an aneuploid egg will result in an embryo with an abnormal number of chromosomes, making it unable to develop into a healthy baby.

Chromosomal abnormalities are the main cause of failed implantation, pregnancy losses, and birth defects. As women get older, the risk of chromosomal abnormalities in embryos rises, leading to lower IVF success rates. Additionally, older women may experience hormonal imbalances that further affect egg quality and development. However, personalized stimulation protocols can help protect egg quality and improve IVF outcomes by regulating hormone production and activity.

When it comes to IVF in older women, selecting the right ovarian stimulation protocol is crucial. Various protocols are available, each tailored to meet individual needs. However, certain protocols should be avoided for older women or those with DOR to optimize chances of success.

I selectively use a variety of ovarian stimulation protocols for ovarian stimulation/IVF in older women and those with DOR :

  • The conventional long pituitary down-regulation protocol: This involves administering a GnRH agonist like Lupron or Buserelin for a few days prior to initiating ovarian stimulation with gonadotropins. Then, a combination of FSH-dominant gonadotropin and a small dose of Menopur is administered, and ultrasound and blood tests are done to monitor follicle development. The eggs are triggered for maturation with hCG, and the egg retrieval is scheduled for approximately 36 hours later. This protocol is often preferred for older women who have adequate ovarian reserve (AMH=>1.5ng/ml).
  • The agonist/antagonist conversion protocol (A/ACP):, This is similar to the conventional long down-regulation protocol. However, instead of using an agonist, a GnRH antagonist is administered from the onset of stimulation with gonadotropins. This protocol is often preferred for older women who have moderately severe DOR (AMH=0.5-1.5ng/ml).
  • A/ACP with Estrogen “priming”; For women with very severe, DORI prescribe  estrogen “priming “with skin estradiol (E2) patches or Estradiol injections administered bi-weekly. For some time before commencing gonadotropin stimulation, in an attempt to enhance ovarian response to stimulation. This protocol is sometimes used in older women who have severe DOR ( <0.5-1.5ng/ml).

In my opinion, the following ovarian stimulation protocols all promote over-exposure to LH-induced ovarian testosterone and are best avoided in older women and women with DOR, undergoing ovarian stimulation for IVF:

  • Agonist “flare” protocols, which cause a surge of pituitary-LH at the wrong time.
  • High dosages , LH-containing fertility drugs (e.g., menotropins such as Menopur).
  • Testosterone-based supplements like Androgel.
  • DHEA supplementation: DHEA is converted to testosterone in the ovaries.
  • Clomiphene citrate & Letrozole, promote exaggerated pituitary LH release that can result in over-production of ovarian testosterone.
  • Triggering egg maturation with too low a dosage of hCG (the ideal dosage is 10,000U of urine derived hCG) andf Recombinant DNA-derived hCG ( the ideal dosage is 500mcg of Ovidrel).

In cases where using their own eggs is no longer viable due to age and severe DOR, using donor eggs provides a fulfilling path to parenthood. Although some may initially hesitate due to the lack of genetic relation, it’s important to understand that the person who gives birth is considered the true biological parent in most cultures and legal systems. Becoming a parent through this connection can bring immense joy and fulfillment, as countless successful cases have shown.

Age may reduce the chances, but it does not eliminate the possibility of having a child through IVF. When IVF with own eggs is not an option, embracing the alternative of egg donation opens doors to highly successful and fulfilling paths to parenthood. It’s time to unlock the possibilities and embark on the journey towards creating a loving family.

__________________________________________________________________

Herewith are  online links to 2  E-books recently  co-authored with  my partner at SFS-NY  (Drew Tortoriello MD)……. for your reading pleasure:

  1. From In Vitro Fertilization to Family: A Journey with Sher Fertility Solutions (SFS) ; https://sherfertilitysolutions.com/sher-fertility-solutions-ebook.pdf

 

  1. Recurrent Pregnancy Loss and Unexplained IVF Failure: The Immunologic Link ;https://drive.google.com/file/d/1iYKz-EkAjMqwMa1ZcufIloRdxnAfDH8L/view

 

I invite you to visit my very recently launched “Podcast”,  “HAVE A BABY” on RUMBLE;   https://rumble.com/c/c-3304480

If you are interested in having an online consultation with me, please contact my assistant, Patti Converse at 702-533-2691 or email her at concierge@sherivf.com\

 

 

 

 

 

 

Name: Jen C

Hello-

I was diagnosed with DOR at age 29 with AMH 0.3 (AMH stable x 3yr, FSH ~12), otherwise no reproductive abnormalities for myself or my husband. Over the past 2yr, I have completed 12 rounds of IVF – the first 6 were poor response with only 1-3 follicles appearing mature on imaging and estrogen levels (medium and high antagonist, flare protocol, and mini IVF), trigger with 10k HCG + lupron and only 2 germinal vesicles retrieved. Next 6 cycles moved to high dose antagonist at a new clinic with 4-9 mature follicles per cycles, trigger with 20k HCG/lupron x2 each retrieval at 36-38hr and 12h later, with 0-3 eggs retrieved, all germinal vesicle stage. Is there anything that can be added? Are further IVF attempts futile? Is timed intercourse futile?

Thank you.

Answer:

_

Empty Follicle Syndrome” is a misleading term because follicles always contain eggs. However, some eggs may have difficulties detaching and being retrieved. This is more likely to happen when multiple attempts are needed to retrieve an egg from a follicle, indicating the egg may have chromosomal abnormalities.

The hormonal environment created during controlled ovarian stimulation plays a significant role in egg development. In certain cases, follicles may not release their eggs during retrieval, leading to the misconception of “empty” follicles.

This situation is most commonly encountered in older women, those with diminished ovarian reserve (DOR), and women with polycystic ovarian syndrome (PCOS). To address this problem, personalized protocols for controlled ovarian stimulation and careful administration of the hCG trigger shot are important.

The hCG trigger shot is given after optimal ovarian stimulation to initiate the process of reducing the number of chromosomes in the egg. It also helps the egg detach from the follicle’s inner wall. This allows for easier retrieval during the egg retrieval procedure.

Women with increased LH activity, such as older women, those with DOR, and women with PCOS, are more susceptible to the negative effects of LH-induced ovarian testosterone. Excessive LH activity can compromise egg development and increase the chances of chromosomal abnormalities. Medications like clomiphene and Letrozole can stimulate LH release, and certain drugs containing LH or hCG can have negative consequences.

Individualizing the controlled ovarian stimulation protocol, determining the correct dosage and type of hCG trigger, and administering it at the right time are crucial. The recommended dosage of urinary-derived hCG products is 10,000 units, while for recombinant DNA-derived hCG, the optimal dosage is 500 micrograms. A lower dosage of hCG can increase the risk of chromosomal abnormalities in the eggs and negatively impact the outcome of IVF.

Understanding the role of LH activity, the effects of medications on hormone release, and the importance of personalized protocols are vital. By optimizing these factors, the risk of failed egg retrieval and “empty follicle syndrome” can be minimized, improving the chances of successful IVF outcomes.

________________________________________________________________________

Herewith are  online links to 2  E-books recently  co-authored with  my partner at SFS-NY  (Drew Tortoriello MD)……. for your reading pleasure:

  1. From In Vitro Fertilization to Family: A Journey with Sher Fertility Solutions (SFS) ; https://sherfertilitysolutions.com/sher-fertility-solutions-ebook.pdf

 

  1. Recurrent Pregnancy Loss and Unexplained IVF Failure: The Immunologic Link ;https://drive.google.com/file/d/1iYKz-EkAjMqwMa1ZcufIloRdxnAfDH8L/view

 

I invite you to visit my very recently launched “Podcast”,  “HAVE A BABY” on RUMBLE;   https://rumble.com/c/c-3304480

If you are interested in having an online consultation with me, please contact my assistant, Patti Converse at 702-533-2691 or email her at concierge@sherivf.com\

 

Name: Olivia D

Hi- I have recurrent unexplained pregnancy loss:

Recently got a positive test and have the following HCG results… The third draws doubling time threw me off a bit but today had a good rise… any insight into what to expect from these numbers?🤞🤞
48.9
123.4
209.7
651.4

Answer:

When it comes to reproduction, humans face challenges compared to other mammals. A significant number of fertilized eggs in humans do not result in live births, with up to 75% failing to develop, and around 30% of pregnancies ending within the first 10 weeks  (first trimester). Recurrent pregnancy loss (RPL) refers to two or more consecutive failed pregnancies, which is relatively rare, affecting less than 5% of women for two losses and only 1% for three or more losses. Understanding the causes of pregnancy loss and finding solutions is crucial for those affected. This article aims to explain the different types of pregnancy loss and shed light on potential causes.

Types of Pregnancy Loss: Pregnancy loss can occur at various stages, leading to different classifications:

  1. Early Pregnancy Loss: Also known as a miscarriage, this typically happens in the first trimester. Early pregnancy losses are usually sporadic, not recurring. In over 70% of cases, these losses are due to chromosomal abnormalities in the embryo, where there are more or fewer than the normal 46 chromosomes. Therefore, they are not likely to be repetitive.
  2. Late Pregnancy Loss: Late pregnancy losses occur after the first trimester (12th week) and are less common (1% of pregnancies). They often result from anatomical abnormalities in the uterus or cervix. Weakness in the cervix, known as cervical incompetence, is a frequent cause. Other factors include developmental abnormalities of the uterus, uterine fibroid tumors, intrauterine growth retardation, placental abruption, premature rupture of membranes, and premature labor.

Causes of Recurrent Pregnancy Loss (RPL): Recurrent pregnancy loss refers to multiple consecutive miscarriages. While chromosomal abnormalities are a leading cause of sporadic early pregnancy losses, RPL cases are mostly attributed to non-chromosomal factors. Some possible causes include:

  1. Uterine Environment Problems: Issues with the uterine environment can prevent a normal embryo from properly implanting and developing. These problems may include inadequate thickening of the uterine lining, irregularities in the uterine cavity (such as polyps, fibroid tumors, scarring, or adenomyosis), hormonal imbalances (progesterone deficiency or luteal phase defects), and deficient blood flow to the uterine lining.
  2. Immunologic Implantation Dysfunction (IID): IID is a significant cause of RPL, contributing to 75% of cases where chromosomally normal embryos fail to implant. It involves the immune system’s response to pregnancy, which can interfere with successful implantation.
  3. Blood Clotting Disorders: Thrombophilia, a hereditary clotting disorder, can disrupt the blood supply to the developing fetus, leading to pregnancy loss.
  4. Genetic and Structural Abnormalities: Genetic abnormalities are rare causes of RPL, while structural chromosomal abnormalities occur infrequently (1%). Unbalanced translocation, where part of one chromosome detaches and fuses with another, can lead to pregnancy loss. Studies also suggest that damaged sperm DNA can negatively impact fetal development and result in miscarriage.

 

IMMUNOLOGIC IMPLANTATION DYSFUNCTION AND RPL:

 

Autoimmune IID: Here an immunologic reaction is produced by the individual to his/her body’s own cellular components. The most common antibodies that form in such situations are APA and antithyroid antibodies (ATA). But it is only when specialized immune cells in the uterine lining, known as cytotoxic lymphocytes (CTL) and natural killer (NK) cells, become activated and start to release an excessive/disproportionate amount of TH-1 cytokines that attack the root system of the embryo, that implantation potential is jeopardized. Diagnosis of such activation requires highly specialized blood test for cytokine activity that can only be performed by a handful of reproductive immunology reference laboratories in the United States. Alloimmune IID, (i.e., where antibodies are formed against antigens derived from another member of the same species), is believed to be a common immunologic cause of recurrent pregnancy loss. Autoimmune IID is often genetically transmitted. Thus, it should not be surprising to learn that it is more likely to exist in women who have a family (or personal) history of primary autoimmune diseases such as lupus erythematosus (LE), scleroderma or autoimmune hypothyroidism (Hashimoto’s disease), autoimmune hyperthyroidism (Grave’s disease), rheumatoid arthritis, etc. Reactionary (secondary) autoimmunity can occur in conjunction with any medical condition associated with widespread tissue damage. One such gynecologic condition is endometriosis. Since autoimmune IID is usually associated with activated NK and T-cells from the outset, it usually results in such very early destruction of the embryo’s root system that the patient does not even recognize that she is pregnant. Accordingly, the condition usually presents as “unexplained infertility” or “unexplained IVF failure” rather than as a miscarriage. Alloimmune IID, on the other hand, usually starts off presenting as unexplained miscarriages (often manifesting as RPL). Over time as NK/T cell activation builds and eventually becomes permanently established the patient often goes from RPL to “infertility” due to failed implantation. RPL is more commonly the consequence of alloimmune rather than autoimmune implantation dysfunction. However, regardless, of whether miscarriage is due to autoimmune or alloimmune implantation dysfunction the final blow to the pregnancy is the result of activated natural killer cells (NKa) and cytotoxic lymphocytes (CTL B) in the uterine lining that damage the developing embryo’s “root system” (trophoblast) so that it can no longer sustain the growing conceptus. This having been said, it is important to note that autoimmune IID is readily amenable to reversal through timely, appropriately administered, selective immunotherapy, and alloimmune IID is not. It is much more difficult to treat successfully, even with the use of immunotherapy. In fact, in some cases the only solution will be to revert to selective immunotherapy plus using donor sperm (provided there is no “match” between the donor’s DQa profile and that of the female recipient) or alternatively to resort to gestational surrogacy.

 

DIAGNOSING THE CAUSE OF RPL.

In the past, women who miscarried were not evaluated thoroughly until they had lost several pregnancies in a row. This was because sporadic miscarriages are most commonly the result of embryo numerical chromosomal irregularities (aneuploidy) and thus not treatable. However, a consecutive series of miscarriages points to a repetitive cause that is non-chromosomal and is potentially remediable. Since RPL is most commonly due to a uterine pathology or immunologic causes that are potentially treatable, it follows that early chromosomal evaluation of products of conception could point to a potentially treatable situation. Thus, I strongly recommend that such testing be done in most cases of miscarriage. Doing so will avoid a great deal of unnecessary heartache for many patients. Establishing the correct diagnosis is the first step toward determining effective treatment for couples with RPL. It results from a problem within the pregnancy itself or within the uterine environment where the pregnancy implants and grows. Diagnostic tests useful in identifying individuals at greater risk for a problem within the pregnancy itself include Karyotyping (chromosome analysis) both prospective parents Assessment of the karyotype of products of conception derived from previous miscarriage specimens Ultrasound examination of the uterine cavity after sterile water is injected or sonohysterogram, fluid ultrasound, etc.) Hysterosalpingogram (dye X-ray test) Hysteroscopic evaluation of the uterine cavity Full hormonal evaluation (estrogen, progesterone, adrenal steroid hormones, thyroid hormones, FSH/LH, etc.) Immunologic testing to include Antiphospholipid antibody (APA) panel Antinuclear antibody (ANA) panel Antithyroid antibody panel (i.e., antithyroglobulin and antimicrosomal antibodies) Reproductive immunophenotype Natural killer cell activity (NKa) assay (i.e., K562 target cell test) Alloimmune testing of both the male and female partners

 

TREATMENT OF RPL

  • Treatment for Anatomic Abnormalities of the Uterus: 

This involves restoration through removal of local lesions such as fibroids, scar tissue, and endometrial polyps or timely insertion of a cervical cerclage (a stitch placed around the neck of the weakened cervix) or the excision of a uterine septum when indicated. Treatment of Thin Uterine Lining: A thin uterine lining has been shown to correlate with compromised pregnancy outcome. Often this will be associated with reduced blood flow to the endometrium. Such decreased blood flow to the uterus can be improved through treatment with sildenafil and possibly aspirin. sildenafil (Viagra) Therapy. Viagra has been used successfully to increase uterine blood flow. However, to be effective it must be administered starting as soon as the period stops up until the day of ovulation and it must be administered vaginally (not orally). Viagra in the form of vaginal suppositories given in the dosage of 25 mg four times a day has been shown to increase uterine blood flow as well as thickness of the uterine lining. To date, we have seen significant improvement of the thickness of the uterine lining in about 70% of women treated. Successful pregnancy resulted in 42% of women who responded to the Viagra. It should be remembered that most of these women had previously experienced repeated IVF failures. Use of Aspirin: This is an anti-prostaglandin that improves blood flow to the endometrium. It is administered at a dosage of 81 mg orally, daily from the beginning of the cycle until ovulation.

 

Treating Immunologic Implantation Dysfunction with Selective Immunotherapy: 

Modalities such as intralipid (IL), intravenous immunoglobulin-G (IVIG),  heparinoids (Lovenox/Clexane), and corticosteroids (dexamethasone, prednisone, prednisolone) can be used in select cases depending on autoimmune or alloimmune dysfunction. The Use of IVF in the Treatment of RPL In the following circumstances, IVF is the preferred option: When in addition to a history of RPL, another standard indication for IVF (e.g., tubal factor, endometriosis, and male factor infertility) is superimposed and in cases where selective immunotherapy is needed to treat an immunologic implantation dysfunction.  The reason for IVF being a preferred approach when immunotherapy is indicated is that in order to be effective, immunotherapy needs to be initiated well before spontaneous or induced ovulation. Given the fact that the anticipated birthrate per cycle of COS with or without IUI is at best about 15%, it follows that short of IVF, to have even a reasonable chance of a live birth, most women with immunologic causes of RPL would need to undergo immunotherapy repeatedly, over consecutive cycles. Conversely, with IVF, the chance of a successful outcome in a single cycle of treatment is several times greater and, because of the attenuated and concentrated time period required for treatment, IVF is far safer and thus represents a more practicable alternative Since embryo aneuploidy is a common cause of miscarriage, the use of preimplantation genetic screening/ testing (PGS/T), with tests such as next generation gene sequencing (NGS), can provide a valuable diagnostic and therapeutic advantage in cases of RPL. PGS/T requires IVF to provide access to embryos for testing. There are a few cases of intractable alloimmune dysfunction due to absolute DQ alpha gene matching ( where there is a complete genotyping match between the male and female partners) where Gestational Surrogacy or use of donor sperm could represent the only viable recourse, other than abandoning treatment altogether and/or resorting to adoption. Other non-immunologic factors such as an intractably thin uterine lining or severe uterine pathology might also warrant that last resort consideration be given to gestational surrogacy. Conclusion:

 

Understanding the causes of pregnancy loss is crucial for individuals experiencing recurrent miscarriages. While chromosomal abnormalities are a common cause of sporadic early pregnancy losses, other factors such as uterine environment problems, immunologic implantation dysfunction, blood clotting disorders, and genetic or structural abnormalities can contribute to recurrent losses. By identifying the underlying cause, healthcare professionals can provide appropriate interventions and support to improve the chances of a successful pregnancy. The good news is that if a couple with RPL is open to all of the diagnostic and treatment options referred to above, a live birthrate of 70%–80% is ultimately achievable.

____________________________________________________________________

Herewith are  online links to 2  E-books recently  co-authored with  my partner at SFS-NY  (Drew Tortoriello MD)……. for your reading pleasure:

  1. From In Vitro Fertilization to Family: A Journey with Sher Fertility Solutions (SFS) ; https://sherfertilitysolutions.com/sher-fertility-solutions-ebook.pdf

 

  1. Recurrent Pregnancy Loss and Unexplained IVF Failure: The Immunologic Link ;https://drive.google.com/file/d/1iYKz-EkAjMqwMa1ZcufIloRdxnAfDH8L/view

 

I invite you to visit my very recently launched “Podcast”,  “HAVE A BABY” on RUMBLE;   https://rumble.com/c/c-3304480

If you are interested in having an online consultation with me, please contact my assistant, Patti Converse at 702-533-2691 or email her at concierge@sherivf.com\

 

Name: Polona tara B

Hi, dr. Sher!
Is it true, that a quality of eggs can be worse in a second cycle, if you do one stimulation after another? My doctor said I shold wait a month for another stimulation because of that. Thank you!

Answer:

No! I do not believe so! In my opinion, the most important determinants are age, ovarian reserve and the protocol used for ovarian stimulation.

The journey of in vitro fertilization can be a rollercoaster of emotions for many patients. Often times they have to face the harsh reality that the number and quality of eggs retrieved has fallen short of their expectations. Then, should fertilization of these eggs not propagate  chromosomally normal (euploid), “competent” embryos suitable for transfer to the uterus, many such patients find themselves in a state of emotional distress. They grapple with the inevitable questions of why this happened and how to prevent it from occurring again in the future. This article aims to delve into these queries, providing insights, rational explanations, and therapeutic options. It is an invitation to explore the light at the end of the tunnel. Readers are urged to carefully absorb the entirety of the article in the hope of finding valuable information and renewed hope.

  • The Importance of Chromosomal Integrity: While sperm quality is an important factor, egg quality is by far the most important when it comes to the generation of embryos that are capable of propagating healthy babies (“competent”). In this regard, chromosomal integrity of the egg and embryo, although it is not the only factor , is certainly the main determinant of such competency.
  • The woman’s age: About two thirds of a woman’s eggs in her twenties or early thirties have the correct number of chromosomes, which is necessary for a healthy pregnancy. As a woman gets older, the percentage of eggs with the right number of chromosomes decreases. By age 40, only about one in every 5-6 eggs is likely to be normal, and by the mid-forties, less than one in ten eggs will be normal.
  • Ovarian Reserve (number of available in the ovaries): A woman is born with all the eggs she will ever have. She starts using these eggs when she begins ovulating during puberty. At first, the eggs are used up quickly, but as she gets older, the number of eggs starts to run out. Her brain and pituitary gland try to stimulate the production of more eggs by increasing the output of Follicle Stimulating Hormone (FSH), but unfortunately, this often doesn’t work. When the number of remaining eggs in her ovaries falls below a certain level (which can be different for each woman), her FSH level rises, and production of the ovarian hormone, AMH decreases. This is the start of diminishing ovarian reserve (DOR). Most women experience the onset of DOR in their late 30s or early 40s, but it can happen earlier for some. The lower the ovarian reserve, the lower the AMH level will be, and the fewer eggs will be available for harvesting during IVF-egg retrieval. In such cases, a higher dosage of fertility drugs might be needed to promote better egg production in future attempts. . On the other hand, higher AMH levels mean more eggs are available, and lower doses of fertility drugs are usually needed. DOR is commonly associated with increased bioactivity of pituitary gland-produced LH. This LH activates production of ovarian male hormones (androgens)…predominantly testosterone by ovarian connective tissue (stroma) . While a small amount of  ovarian testosterone is absolutely necessary for optimal follicle and egg development, excessive ovarian testosterone will often access the follicle , and compromise both egg quality and follicle growth and development. In some cases, rapidly increasing  LH-release (“premature LH-surge”) with excessive induced ovarian testosterone can lead to “premature luteinization”  of the follicles with cessation in growth and even to“ premature ovulation”.
  • Importance of Individualized Controlled Ovarian Stimulation (COS) Protocol: It’s not surprising that DOR is more common in older women, but regardless of age, having DOR makes a woman’s eggs more likely to be compromised during controlled ovarian stimulation (COS). The choice of the COS protocol is crucial to preventing unintentional harm to egg and embryo quality. The wrong protocol can disrupt normal egg development and increase the risk of abnormal embryos. That’s why it’s important to tailor the COS protocol to each individual’s needs. This helps optimize follicle growth and the quality of eggs and embryos. The timing of certain treatments is also important for successful outcomes.
  • Embryo Competency and Blastocyst Development: Embryos that don’t develop into blastocysts by day 6 after fertilization are usually chromosomally abnormal or aneuploid (”incompetent”) and not suitable for transfer. However, not all blastocysts are guaranteed to be normal and capable of developing into a healthy baby. As a woman gets older, the chances of a her embryos being chromosomally normal blastocyst decreases. For example, a blastocyst from a 30-year-old woman is more likely to be normal compared to one from a 40-year-old woman.

The IVF stimulation protocol has a big impact on the quality of eggs and embryos especially in women with DOR. Unfortunately, many IVF doctors use the same COS “recipe approach” for everyone without considering individual differences. Using personalized protocols can greatly improve the success of IVF. While we can’t change genetics or reverse a woman’s age, a skilled IVF specialist can customize the COS protocol to meet each patient’s specific needs.

GONADOTROPIN RELEASING HORMONE AGONISTS (GNRHA) AND GNRH-ANTAGONISTS:

  • Gonadotropin releasing hormone agonists (GnRHa). Examples are Lupron, Buserelin, Superfact, and Decapeptyl . These are commonly used to launch  ovarian stimulation cycles. They work by initially causing a release of pituitary gonadotropins, followed by a decrease in LH and FSH levels within 4-7 days. This creates a relatively low LH environment when COS begins, which is generally beneficial for egg development. However, if GnRHa are administered starting concomitant with gonadotropin stimulation (see GnRHa –“flare protocol” -below) it can cause an immediate surge in LH release, potentially leading to high levels of ovarian testosterone that can harm egg quality, especially in older women and those with diminished ovarian reserve (DOR).
  • Gonadotropin releasing hormone antagonists (GnRH-antagonists) : Examples are Ganirelix, Cetrotide, and Orgalutron. GnRH antagonists (take days work quickly (within hours) to block pituitary LH release. Their purpose is to prevent excessive LH release during COS. In contrast, the LH-lowering effect of GnRH agonists takes several days to develop. Traditionally, GnRH antagonists are given starting on the 5th-7th day of gonadotropin stimulation. However, in older women and those with DOR, suppressing LH might happen too late to prevent excessive ovarian androgen production that can negatively impact egg development in the early stages of stimulation. That’s why I prefer to administer GnRH-antagonists right from the beginning of gonadotropin administration.

 

USING BIRTH CONTROL PILLS TO START OVARIAN STIMULATION:

Patients are often told that using birth control pills (BCP) to begin ovarian stimulation will suppress the response of the ovaries. This is true, but only if the BCP is not used correctly. Here’s the explanation:

In natural menstrual cycles and cycles stimulated with fertility drugs, the follicles in the ovaries need to develop receptors that respond to follicle-stimulating hormone (FSH) in order to properly respond to FSH stimulation. Pre-antral follicles (PAFs) do not have these receptors and cannot respond to FSH stimulation. The development of FSH responsivity requires exposure of the pre-antral follicles to FSH for several days, during which they become antral follicles (AFs) and gain the ability to respond to FSH-gonadotropin stimulation. In regular menstrual cycles, the rising FSH levels naturally convert PAFs to AFs. However, the combined BCP suppresses FSH. To counter this suppression, we need to promote increased  FSH production several days before starting COS. This allows the orderly conversion from PAFs to AFs, ensuring proper follicle and egg development.

GnRHa causes an immediate surge in FSH release by the pituitary gland, promoting the conversion from PAF to AF. Therefore, when women take the BCP control pill to launch a cycle of COS, they need to overlap the BCP with a GnRHa for a few days before menstruation. This allows the early recruited PAFs to complete their development and reach the AF stage, so they can respond appropriately to ovarian stimulation. By adjusting the length of time, the woman is on the birth control pill, we can regulate and control the timing of the IVF treatment cycle. Without this step, initiating ovarian stimulation in women coming off birth control pills would be suboptimal.

PROTOCOLS FOR CONTROLLED OVARIAN STIMULATION (COS):

  • GnRH Agonist Ovarian Stimulation Protocols:
    • The long GnRHa protocol: Here, a GnRHa (usually Lupron or Superfact) is given either in a natural cycle, starting 5-7 days before menstruation, overlapping with the BCP for three days. Thereupon, the pill is stopped, while daily  GnRHa injections continue until menstruation occurs (usually 5-7 days later). The GnRHa causes a rapid rise in FSH and LH levels. This is followed about 3-4 days later , by a progressive decline in FSH and LH to near zero levels,  with a concomitant drop in ovarian estradiol and progesterone. This, in turn triggers uterine withdrawal bleeding (menstruation) within 5-7 days of starting the GnRHa administration. Gonadotropin treatment is then initiated while daily GnRHa injections continue to maintain a relatively low LH environment. Gonadotropin administration continues until the hCG “trigger” (see below).
    • Short GnRH-Agonist (“Flare”) Protocol: This protocol involves starting hormone therapy and using GnRH agonist at the same time. The goal is to boost FSH so that with concomitant stimulation with FSH-gonadotropins + the GnRHa-induced surge in pituitary gland FSH release, will augment follicle development. However, this surge also leads to a rise in LH levels, which can cause an excessive production of ovarian male hormones (e.g., testosterone). This could potentially adversely affect the quality of eggs, especially in women over 39 years old, those with low ovarian reserve, and women with PCOS or DOR who already have increased LH sensitivity. In this way, these “flare protocols” can potentially decrease the success rates of IVF. While they are generally safe for younger women with normal ovarian reserve, I personally avoid using this approach on the off chance that even patients with normal ovarian reserve, might experience poor egg quality.
  • GnRH Antagonist-Ovarian Stimulation Protocols:
    • Conventional GnRH Antagonist Protocol: In this approach, daily GnRH antagonist injections are given from the 5th to the 8th day of COS with gonadotropins to the day of the “trigger” (see below). Accordingly, although rapidly acting to lower LH , this effect of GnRH- antagonist only starts suppressing LH from well into the COS cycle which means the ovarian follicles are left exposed and unshielded from pituitary gland -produced, (endogenous) LH during the first several days of stimulation. This can be harmful, especially in the early stage of COS when eggs and follicles are most vulnerable to the effects of over-produced LH-induced excessive ovarian testosterone. Therefore, I believe the Conventional GnRH Antagonist Protocol is not ideal for older women, those with low ovarian reserve, and women with PCOS who already have elevated LH activity. However, this protocol is acceptable for younger women with normal ovarian reserve, although I personally avoid using this approach on the off chance that even patients with normal ovarian reserve, might experience poor egg quality.

It’s important to note that the main reason for using GnRH antagonists is to prevent a premature LH surge, which is associated with poor egg and embryo quality due to follicular exhaustion. However, calling it a “premature LH surge” is misleading because it actually represents the culmination of a progressive increase in LH-induced ovarian testosterone. A better term would be “premature luteinization”. In some such cases, the rise in LH can precipitate “premature ovulation”.

 

  • Agonist/Antagonist Conversion Protocol (A/ACP): I recommend this protocol for many of my patients, especially for older women and those with DOR or PCOS. The woman starts by taking a BCP for 7-10 days. This overlapped with a GnRHa for 3 days and continued until menstruation ensues about 5-7 days later. At this point  she “converts” from the GnRH-agonist to a GnRH-antagonist (Ganirelix, Orgalutron, or Cetrotide). A few days after this conversion from agonist to antagonist, COS with  gonadotropin stimulation starts. Both the antagonist and the gonadotropins are continued together until the hCG trigger. The purpose is to suppress endogenous LH release throughout the COS process and so  avoid over-exposure of follicles and eggs to LH-induced  excessive ovarian testosterone which as previously stated, can compromise egg and follicle growth and development.   Excessive ovarian testosterone can also adversely affect estrogen-induced growth of the uterine lining (endometrium). Unlike GnRH-agonists, antagonists do not suppress ovarian response to the gonadotropin stimulation. This is why the A/ACP is well-suited for older women and those with diminished ovarian reserve.
  • A/ACP with estrogen priming: This is a modified version of the A/ACP protocol used for women with very low ovarian reserve (AMH=<0.2ng/ml). Estrogen priming is believed to enhance the response of follicles to gonadotropins. Patients start their treatment cycle by taking a combined birth control pill (BCP) for 7-10 days. After that, they overlap daily administration of a GnRH agonist with the BCP for 3 days. The BCP is then stopped, and the daily agonist continues until menstruation ensues (usually 5-7 days later). At this point, the GnRH agonist is supplanted by daily injections of  GnRH antagonist and  Estradiol (E2) “priming” begins using E2 skin patches or intramuscular estradiol valerate injections twice weekly, while continuing the GnRH antagonist. Seven days after starting the estrogen priming COS begins using recombinant FSHr such as Follistim, Gonal-F or Puregon) +menotropin (e.g., Menopur) . The estrogen “priming” continues to the day of the “trigger” (see below).  Egg retrieval is performed 36 hours after the trigger.


Younger women (under 30 years) and women with absent, irregular, or dysfunctional ovulation, as well as those with polycystic ovarian syndrome (PCOS), are at risk of developing a severe condition called Ovarian Hyperstimulation Syndrome (OHSS), which can be life-threatening. To predict this condition, accurate daily blood E2 level monitoring is required.

 

TRIGGERING “EGG MATURATION PRIOR TO EGG RETRIEVAL”

  • The hCG “trigger”: When it comes to helping eggs mature before retrieval, one of the important decisions the doctor needs to make is choosing the “trigger shot” to facilitate the process. Traditionally, hCG (human chorionic gonadotropin) is derived from the urine of pregnant women (hCGu) while a newer recombinant hCG (hCGr), Ovidrel was recently The ideal dosage of hCGu is 10,000U and for Ovidrel, the recommended dosage is 500mcg. Both have the same efficacy. The “trigger” is usually administered by intramuscular injection, 34-36 hours prior to egg retrieval.

Some doctors may choose to lower the dosage of hCG if there is a risk of severe ovarian hyperstimulation syndrome (OHSS). However, I believe that a low dose of hCG (e.g., 5000 units of hCGu or 250 mcg of hCGr ( Ovidrel) might not be enough to optimize egg maturation, especially when there are many follicles. Instead, I suggest using a method called “prolonged coasting” to reduce the risk of OHSS.

  • Using GnRH antagonist alone or combined with hCG as the trigger: Some doctors may prefer to use a GnRH- agonist trigger instead of hCG to reduce the risk of OHSS. The GnRHa “trigger” acts by inducing a “surge of pituitary gland-LH. However, it is difficult to predict the amount of LH that is released in response to a standard agonist trigger. In my opinion, using hCG is a better choice, even in cases of ovarian hyperstimulation, with the condition that “prolonged coasting” is implemented beforehand.
  • Combined use of hCG + GnRH agonist: This approach is better than using a GnRH agonist alone but still not as effective as using the appropriate dosage of hCG.
  • Timing of the trigger: The trigger shot should be given when the majority of ovarian follicles have reached a size of more than 15 mm, with several follicles measuring 18-22 mm. Follicles larger than 22 mm often contain overdeveloped eggs, while follicles smaller than 15 mm usually have underdeveloped and potentially abnormal eggs.

SEVERE OVARIAN HYPERSTIMULATION SYNDROME (OHSS) & “PROLONGED COASTING”

OHSS is a life-threatening condition that can occur during controlled ovarian stimulation (COS) when the blood E2 (estradiol) level rises too high. It is more common in young women with high ovarian reserve, women with polycystic ovarian syndrome (PCOS), and young women who do not ovulate spontaneously. To prevent OHSS, some doctors may trigger egg maturation earlier, use a lower dosage of hCG, or “trigger” using a GnRHa. However, these approaches can compromise egg and embryo quality and reduce the chances of success.

To protect against the risk of OHSS while optimizing egg quality, Physicians can use one of two options. The first is “prolonged coasting,” a procedure I introduced more than three decades ago. It involves stopping gonadotropin therapy while continuing to administer the GnRHa until the risk of OHSS has decreased. The precise timing of “prolonged coasting” is critical. It should be initiated when follicles have reached a specific size accompanied and the  blood estradiol has reached a certain peak.  The second option is to avoid fresh embryo transfer and freeze all “competent” embryos for later frozen embryo transfers (FETs) at a time when the risk of OHSS has subsided. By implementing these strategies, both egg/embryo quality and maternal well-being can be maximized.

 

In the journey of fertility, a woman is blessed with a limited number of eggs, like precious treasures awaiting their time. As she blossoms into womanhood, these eggs are gradually used, and the reserves start to fade. Yet, the power of hope and science intertwines, as we strive to support the development of these eggs through personalized treatment. We recognize that each woman is unique, and tailoring the protocol to her individual needs can unlock the path to success. We embrace the delicate timing, understanding that not all embryos are destined for greatness. With age, the odds may shift, but our dedication remains steadfast, along with our ultimate objective, which is  to do everything possible to propagate  of a normal pregnancy while optimizing the  quality of that life after birth and all times, minimizing risk to the prospective parents.

 _______________________________________________________________________

Herewith are  online links to 2  E-books recently  co-authored with  my partner at SFS-NY  (Drew Tortoriello MD)……. for your reading pleasure:

  1. From In Vitro Fertilization to Family: A Journey with Sher Fertility Solutions (SFS) ; https://sherfertilitysolutions.com/sher-fertility-solutions-ebook.pdf

 

  1. Recurrent Pregnancy Loss and Unexplained IVF Failure: The Immunologic Link ;https://drive.google.com/file/d/1iYKz-EkAjMqwMa1ZcufIloRdxnAfDH8L/view

 

I invite you to visit my very recently launched “Podcast”,  “HAVE A BABY” on RUMBLE;   https://rumble.com/c/c-3304480

If you are interested in having an online consultation with me, please contact my assistant, Patti Converse at 702-533-2691 or email her at concierge@sherivf.com\

 

Ya

Name: Saylin n

Es necesario tener seguro

Answer:

Please post in English!

Geoff Sher

Name: Michelle R

Hello. I am 45 and I am days away from medicated FET. I did a mock FET first, and then I was cancelled during my last FET round (right at the end) because I ovulated. So I waited, scheduled a few FET, and they gave me Lupron this time. Well, in a true moment of “IVF,” with all there is to remember, I did my first PIO injection in hurry, and made it ten minutes late for the exact time on my protocol. The nurse said since it was so close, they would not cancel me. After-hours last night as I checked protocol for the next day (today) I realized I took 25 mg of progesterone (I read it has .25 ml) instead of 50 mg, or .5 ml. So I took the missed amount many hours later. This round has gone beautifully. My health and mindset are dialed. I have taken every single supplement, Lupron injection, viagra, and estradiol like clockwork — never missed a thing, and I am killing myself over this small mistake. Uterine lining, and levels are great. This is our only embryo (tested among 3 blasts from my first ER). I love my clinic, but I feel strongly if I alert them to this small shortage, they will cancel my cycle. The spending has been as high as you can imagine. Many cancellations of subsequent ER attempts, the last FET cancellation, the mock, etc. Can I truly impact my entire cycle from 12.5 mg of PIO? I am set to begin 1.5 Ml daily (no vaginal inserts) today. Day 2. With my FET scheduled for Tuesday the 14th. That is 5 days and a few hours from now. Thank you so much!

Answer:

In my opinion, your errors were minor and in any case, you apparently corrected for them in a timely manner.

I do not think you have compromised your chances at all!

Good luck!

Geoff Sher

Name: Liz C

s ago. Hi! I read about an IVF alternative called CAPA-IVM from this article: https://www.sciencealert.com/ivf-alternative-first-baby-born-in-australia-through-cheaper-safer-fertility-treatment

I want to know if you offer this procedure or, if not, can you recommend a place that does it in the US? Thanks!

Answer:

CAPA-IVM is Biphasic In-Vitro Maturation. of eggs. It is a technical variation on conventional maturation of early eggs (in vitro maturation) that was introduced in Australia about 4y ago.. It has not taken hold in this country as it does not offer any real advantage , in my opinion. In fact, “conventional IVM” has also lost favor here.

I do not know who offers it here . You will nee3de to ask around.

 

Geoff Sher

________________________________________________________________________________________________

Herewith are  online links to 2  E-books recently  co-authored with  my partner at SFS-NY  (Drew Tortoriello MD)……. for your reading pleasure:

  1. From In Vitro Fertilization to Family: A Journey with Sher Fertility Solutions (SFS) ; https://sherfertilitysolutions.com/sher-fertility-solutions-ebook.pdf

 

  1. Recurrent Pregnancy Loss and Unexplained IVF Failure: The Immunologic Link ;https://drive.google.com/file/d/1iYKz-EkAjMqwMa1ZcufIloRdxnAfDH8L/view

 

I invite you to visit my very recently launched “Podcast”,  “HAVE A BABY” on RUMBLE;   https://rumble.com/c/c-3304480

If you are interested in having an online consultation with me, please contact my assistant, Patti Converse at 702-533-2691 or email her at concierge@sherivf.com\

 

Name: Okeoma N

When is best time to start and stop Viagra through vigaina.

Answer:

THE IMPACT OF A THIN UTERINE LINING ON EMBRYO IMPLANTATION: THE BENEFITS OF VIAGRA THERAPY

Geoffrey Sher MD

Back in 1989, I conducted a study that examined how the thickness of a woman’s uterine lining, known as the endometrium, affected the successful implantation of embryos in IVF patients. The study revealed that when the uterine lining measured less than 8mm in thickness by the day of the “hCG trigger” in fresh IVF cycles, or at the start of progesterone therapy in embryo recipient cycles (such as frozen embryo transfers or egg donation IVF), the chances of pregnancy and birth were significantly improved. In my opinion, an ideal estrogen-promoted endometrial lining should measure at least 9mm in thickness, while a lining of 8-9mm is considered “intermediate.” In most cases, an estrogenic lining of less than 8mm is unlikely to result in a viable pregnancy.

A “poor” uterine lining typically occurs when the innermost layer of the endometrium, called the basal or germinal endometrium, fails to respond to estrogen and cannot develop a thick enough outer “functional” layer to support optimal embryo implantation and placenta development. The “functional” layer makes up two-thirds of the total endometrial thickness and is the layer that sheds during menstruation if no pregnancy occurs.

The main causes of a “poor” uterine lining include:

  1. Damage to the basal endometrium due to:
    • Inflammation of the endometrium (endometritis) often resulting from retained products of conception after abortion, miscarriage, or birth.
    • Surgical trauma caused by aggressive uterine scraping during procedures like D&C.
  1. Insensitivity of the basal endometrium to estrogen due to:
    • Prolonged or excessive use of clomiphene citrate.
    • Prenatal exposure to diethylstilbestrol (DES), a drug given to pregnant women in the 1960s to prevent miscarriage.
  1. Overexposure of the uterine lining to ovarian male hormones, mainly testosterone, which can occur in older women, women with diminished ovarian reserve, and women with polycystic ovarian syndrome (PCOS) who have increased LH biological activity. This hormonal imbalance leads to the overproduction of testosterone in the ovary’s connective tissue, further exacerbated by certain ovarian stimulation methods used in IVF.
  2. Reduced blood flow to the basal endometrium, often caused by:
    • Multiple uterine fibroids, especially those located beneath the endometrium (submucosal).
    • Uterine adenomyosis, an abnormal invasion of endometrial glands into the uterine muscle.

“The Viagra Connection”

Eighteen years ago, I reported on the successful use of vaginal Sildenafil (Viagra) in treating women with implantation dysfunction caused by thin endometrial linings. This breakthrough led to the birth of the world’s first “Viagra baby.” Since then, thousands of women with thin uterine linings have been treated with Viagra, and many have gone on to have babies after multiple unsuccessful IVF attempts.

Viagra gained popularity in the 1990s as an oral treatment for erectile dysfunction. Inspired by its mechanism of action, which increases penile blood flow through enhanced nitric oxide activity, I investigated whether vaginal administration of Viagra could improve uterine blood flow, deliver more estrogen to the basal endometrium, and promote endometrial thickening. Our findings confirmed that vaginal Viagra achieved these effects, while oral administration did not provide significant benefits. To facilitate treatment, we collaborated with a compound pharmacy to produce vaginal Viagra suppositories.

In our initial trial, four women with a history of poor endometrial development and failed conception underwent IVF treatment combined with vaginal Viagra therapy. The Viagra suppositories were administered four times daily for 8-11 days and stopped 5-7 days before embryo transfer. This treatment resulted in a rapid and significant improvement in uterine blood flow, leading to enhanced endometrial development in all four cases. Three of these women subsequently conceived. In 2002, I expanded the trial to include 105 women with repeated IVF failure due to persistently thin endometrial linings. About 70% of these women responded positively to Viagra therapy, with a notable increase in endometrial thickness. Forty-five percent achieved live births after a single cycle of IVF with Viagra treatment, and the miscarriage rate was only 9%. Women who did not show improvement in endometrial thickness following Viagra treatment did not achieve viable pregnancies.

When administered vaginally, Viagra is quickly absorbed and reaches the uterine blood system in high concentrations. It then dilutes as it enters the systemic circulation, explaining why treatment is virtually free from systemic side effects.

It is important to note that Viagra may not improve endometrial thickness in all cases. Approximately 30-40% of women treated may not experience any improvement. In severe cases of thin uterine linings where the basal endometrium has been permanently damaged and becomes unresponsive to estrogen, Viagra treatment is unlikely to be effective. This can occur due to conditions such as post-pregnancy endometritis, chronic inflammation resulting from uterine tuberculosis (rare in the United States), or extensive surgical damage to the basal endometrium.

In my practice, I sometimes recommend combining vaginal Viagra administration with oral Terbutaline (5mg). Viagra relaxes the muscle walls of uterine spiral arteries, while terbutaline relaxes the uterine muscle itself. The combination of these medications synergistically enhances blood flow through the uterus, improving estrogen delivery to the endometrial lining. However, it’s important to monitor potential side effects of Terbutaline such as agitation, tremors, and palpitations. Women with cardiac disease or irregular heartbeat should not use Terbutaline.

Approximately 75% of women with thin uterine linings respond positively to treatment within 2-3 days. Those who do not respond well often have severe inner ( (basal) endometrial lining damage, where improved uterine blood flow cannot stimulate a positive response. Such cases are commonly associated with previous pregnancy-related endometrial inflammation, occurring after abortions, infected vaginal deliveries, or cesarean sections.

Viagra therapy has been a game-changer for thousands of women with thin uterine linings, allowing them to successfully overcome infertility and build their families.

 ____________________________________________________________

PLEASE SHARE THIS WITH OTHERS AND HELP SPREAD THE WORD!!

 

Herewith are  online links to 2  E-books recently  co-authored with  my partner at SFS-NY  (Drew Tortoriello MD)……. for your reading pleasure:

  1. From In Vitro Fertilization to Family: A Journey with Sher Fertility Solutions (SFS) ; https://sherfertilitysolutions.com/sher-fertility-solutions-ebook.pdf

 

  1. Recurrent Pregnancy Loss and Unexplained IVF Failure: The Immunologic Link ;https://drive.google.com/file/d/1iYKz-EkAjMqwMa1ZcufIloRdxnAfDH8L/view

 

I invite you to visit my very recently launched “Podcast”,  “HAVE A BABY” on RUMBLE;   https://rumble.com/c/c-3304480

If you are interested in having an online consultation with me, please contact my assistant, Patti Converse at 702-533-2691 or email her at concierge@sherivf.com\

 

Name: Nomonde M

I don’t seem to understand these hcg levels. I did a blood test the first time and it showed that my hcg levels were on 6105 and I am 5 weeks pregnant. I did a test again a week after and it shows 1501 hcg levels and my nurse says I am 6 weeks 2 days pregnant. Exactly what could it be because isn’t the number supposed to be higher at 6 weeks

Answer:

Ideally the level should have increased. Repeat the hCG again.

Geoff Sher

TIMING AND INTERPRETATION OF hCG BLOOD PREGNANCY TESTS

Geoffrey Sher MD

Going through IVF is a major investment, emotionally, physically, and financially, for every patient or couple. One of the most crucial moments is receiving the result of the blood test for human chorionic gonadotropin (hCG) pregnancy. It’s a big deal! The days after the embryo transfer, waiting for this result, can be extremely stressful. That’s why it’s crucial for the IVF doctor and staff to handle this information with care and professionalism. They should be accessible to the patient/couple and provide results promptly and sensitively.

Testing urine or blood to check for human chorionic gonadotropin (hCG) is the best way to confirm pregnancy. Urine tests are cheaper and more commonly used. They are also more convenient because they can be done anywhere. However, blood tests are more reliable and sensitive than urine tests. They can detect pregnancy earlier and at lower hCG levels. Blood tests are also more accurate and can track changes in hCG levels over time. Urine tests can detect hCG when blood levels are above 20IU, which is about 16-18 days after ovulation or 2-3 days after a missed period. Blood tests can measure any concentration of hCG about 12-13 days after ovulation.

Detecting hCG in the blood early on and tracking its increase is especially useful for women undergoing fertility treatments like controlled ovarian stimulation or in vitro fertilization. The sooner hCG is detected and measured, the more information can be gathered about the success of implantation and the health of the developing embryo.

Typically, two beta hCG blood tests are done, spaced 2-4 days apart. It’s best to wait for the results of the second test before reporting on the pregnancy. This is because an initial result can change, even from equivocal or negative to positive. Sometimes a normal embryo takes longer to implant, and the hCG level can be initially low or undetectable. Regardless of the initial level, the test should be repeated after two days to check for a significant rise in hCG. A significant rise usually indicates that an embryo is implanting, which suggests a possible pregnancy. Waiting for the second test result helps avoid conveying false hope or disappointment.


It’s important to note that beta hCG levels don’t double every two days throughout pregnancy. Once the levels rise above 4,000U, they tend to increase more slowly. Except in specific cases like IVF using an egg donor or transfer of genetically tested embryos, the birth rate following IVF in younger women is around 40% per embryo transfer. Patients need to have realistic expectations and should be informed about how and when they will receive the news, as well as counseling in case of a negative outcome.

When an embryo starts to implant, it releases the pregnancy hormone hCG into the woman’s bloodstream. Around 12 days after egg retrieval, 9 days after a day 3 embryo transfer, or 7 days after a blastocyst transfer, a woman should have a quantitative beta hCG blood pregnancy test performed. By that time, most of the hCG injected to prepare the eggs for retrieval should have cleared from the bloodstream. So, if the test detects more than 10 IU of hCG per ml of blood, it indicates that the embryo has attempted to implant. In third-party IVF (e.g., ovum donation, gestational surrogacy, embryo adoption, or frozen embryo transfers), no hCG trigger is administered, so any amount of hCG detected in the blood is considered significant.

Sometimes, there is a slow initial rise in hCG between the first and second tests (failure to double every 48 hours). In such cases, a third and sometimes a fourth hCG test should be done at two-day intervals. A failure to double on the third and/or fourth test is a poor sign and could indicate a failed or dysfunctional implantation. In some cases, a progressively slow rising hCG level might indicate an ectopic pregnancy, which requires additional testing and follow-up.

In certain situations, the first beta hCG level starts high, drops with the second test, and then starts doubling again. This could suggest that initially, multiple embryos started to implant but only one survived to continue a healthy implantation.

It’s customary for the IVF clinic staff to inform the patient/couple and the referring physician about the hCG pregnancy test results. Often, the IVF physician or nurse-coordinator coordinates with the referring physician to arrange all necessary pregnancy tests. If the patient/couple prefer to make their own arrangements, the program should provide detailed instructions.

In some cases, when the two blood pregnancy tests show that one or more embryos are implanting, certain programs recommend daily injections of progesterone or the use of vaginal hormone suppositories for several weeks to support the implantation process. Others give hCG injections three times a week until the pregnancy can be confirmed by ultrasound examination. Some IVF programs don’t prescribe any hormones after the embryo transfer.

Patients with appropriate doubling of hCG levels within two days after frozen embryo transfer (FET) or third-party IVF procedures such as surrogacy or egg donation may receive estradiol and progesterone injections, often along with vaginal hormone suppositories, for 10 weeks after the implantation is diagnosed by blood pregnancy testing.

A positive Beta hCG blood pregnancy test indicates the possibility of conception, but ultrasound confirmation is needed to confirm the pregnancy. Until then, it is referred to as a “chemical pregnancy.” Only when ultrasound examination confirms the presence of a gestational sac, clinical examination establishes a viable pregnancy, or after abortion when products of conception are detected, is it called a clinical intrauterine pregnancy.

A significantly elevated  hCG blood level without concomitant detection of an gestational sac inside the uterus by ultrasound after 5 weeks gestation raises the suspicion of an ectopic (tubal) pregnancy.

The risk of miscarriage gradually decreases once a viable clinical pregnancy is diagnosed (a conceptus with a regular heartbeat of 110-180 beats per minute). From this point onward, the risk of miscarriage is usually 10- 15% for women under 40 years old and around 35% for women in their early forties.

Dealing with successful IVF cases is relatively easy as everyone feels happy and validated. The real challenge lies in handling unsuccessful cases. Setting rational expectations from the beginning is crucial. In some cases (fortunately rare), emotional pressure may overwhelm the patient/couple, leading to a need for counseling or psychiatric therapy. I always advise my patients that receiving optimal care doesn’t always guarantee the desired outcome. There are many variables beyond our control, especially the unpredictable nature of fate. With around 36 years of experience in this field, I strongly believe that when it comes to IVF, the saying “man proposes while God disposes” always holds.

There are a few important things to consider when interpreting blood hCG levels. Levels can vary widely, ranging from 5mIU/ml to over 400mIU/ml, 10 days after ovulation or egg retrieval. The levels double every 48-72 hours until the 6th week of pregnancy, after which the doubling rate slows down to about 96 hours. By the end of the 1st trimester, hCG levels reach 13,000-290,000 IU and then slowly decline to around 26,000-300,000 IU at full term. Here are the average hCG levels during the first trimester:

  • 3 weeks after the last menstrual period (LMP): 5-50 IU
  • 4 weeks LMP: 5-426 IU
  • 5 weeks LMP: 18-7,340 IU
  • 6 weeks LMP: 1,080-56,500 IU
  • 7-8 weeks LMP: 7,650-229,000 IU
  • 9-12 weeks LMP: 25,700-288,000 IU

Most doctors wait until around the 7th week to perform an ultrasound to confirm pregnancy. By that time, the heartbeat should be clearly visible, providing a more reliable assessment of the pregnancy’s viability.

In some cases, blood hCG levels can be unusually high or increase faster than normal. This could indicate multiple pregnancies or a molar pregnancy. Rarely, conditions unrelated to pregnancy, such as certain ovarian tumors or cancers, can cause detectable hCG levels in both blood and urine.

 

To summarize, testing urine or blood for hCG is the most reliable way to confirm pregnancy. Urine tests are more common and convenient, while blood tests are more accurate and can detect pregnancy earlier. Tracking hCG levels in the blood is especially important for women undergoing fertility treatments. It’s essential to wait for the results of a second blood test before confirming pregnancy to avoid false hope or disappointment. Interpreting hCG levels requires considering various factors, and doctors usually perform an ultrasound around the 7th week for a more accurate assessment. Unusually high hCG levels may indicate multiple pregnancies or other conditions unrelated to pregnancy. Providing sensitive and timely communication of results is crucial for IVF clinics to support patients through the emotional journey.

______________________________________________________

Herewith are  online links to 2  E-books recently  co-authored with  my partner at SFS-NY  (Drew Tortoriello MD)……. for your reading pleasure:

  1. From In Vitro Fertilization to Family: A Journey with Sher Fertility Solutions (SFS) ; https://sherfertilitysolutions.com/sher-fertility-solutions-ebook.pdf

 

  1. Recurrent Pregnancy Loss and Unexplained IVF Failure: The Immunologic Link ;https://drive.google.com/file/d/1iYKz-EkAjMqwMa1ZcufIloRdxnAfDH8L/view

 

I invite you to visit my very recently launched “Podcast”,  “HAVE A BABY” on RUMBLE;   https://rumble.com/c/c-3304480

If you are interested in having an online consultation with me, please contact my assistant, Patti Converse at 702-533-2691 or email her at concierge@sherivf.com\

 

Name: Nora R

Hace 7 años me hice una ligadura de trompas con mi última hija en mi país. Tengo 2 niñas. Ahora deseo tener un bebé. Tengo 30 años. Quiero saber el precio de la reconstrucción de trompas.

Answer:

Please post your question in English!

 

Geoff Sher

_________________________________________________________________________________

PLEASE SHARE THIS WITH OTHERS AND HELP SPREAD THE WORD!!

 

Herewith are  online links to 2  E-books recently  co-authored with  my partner at SFS-NY  (Drew Tortoriello MD)……. for your reading pleasure:

  1. From In Vitro Fertilization to Family: A Journey with Sher Fertility Solutions (SFS) ; https://sherfertilitysolutions.com/sher-fertility-solutions-ebook.pdf

 

  1. Recurrent Pregnancy Loss and Unexplained IVF Failure: The Immunologic Link ;https://drive.google.com/file/d/1iYKz-EkAjMqwMa1ZcufIloRdxnAfDH8L/view

 

I invite you to visit my very recently launched “Podcast”,  “HAVE A BABY” on RUMBLE;   https://rumble.com/c/c-3304480

If you are interested in having an online consultation with me, please contact my assistant, Patti Converse at 702-533-2691 or email her at concierge@sherivf.com\

 

Name: Julie A

I just Received blood work where my dhea levels are elevated and they might believe it’s associate with pcos. What can I do In regards to having a baby?

Answer:

Raised DHEA is not a real problem . it is a raised DHEAS that xcouyld be problematic as it would point to a possible adrenal contribution to the PCOS equation.

Navigating Polycystic Ovary Syndrome: Understanding, Hope, and Treatment

 

Geoffrey Sher MD

 

 

Understanding the intricate interplay of hormones and the impact on egg development empowers us to create personalized protocols, offering hope for improved egg quality and ultimately optimizing the chances of successful IVF for women with PCOS.

 

 

 

Polycystic ovary syndrome (PCOS) is a widespread hormonal disorder affecting 5% to 10% of reproductive-age women globally. Women with PCOS often have enlarged ovaries containing multiple small fluid-filled collections (micro-cysts) arranged in a “string of pearls” pattern below the ovarian surface, intertwined with an overgrowth of ovarian connective tissue.

 

PCOS is marked by abnormal ovarian function causing absent, irregular or dysfunctional ovulation and menstruation,  infertility, increased body hair (hirsutism), acne, and higher body weight as indicated by an above normal body mass index (BMI). 

 

Despite substantial research efforts to identify its cause, the origins of PCOS remain elusive, and a definite cure is yet to be found. This disorder is notably diverse and often has a genetic basis within families. 

 

Infertility related to PCOS is attributed to various factors, including irregular gonadotropin (FSH and LH) pituitary secretion, peripheral insulin resistance, elevated levels of adrenal and/or ovarian androgens (male hormones), and dysfunction in growth factors. Individuals with PCOS often battle obesity and insulin resistance. The compensatory surge in insulin levels further stimulates ovarian androgen production, potentially hampering egg maturation. Notably, the degree of insulin resistance is closely linked to anovulation. 

 

PCOS also poses long-term health risks, underscoring the need for vigilant annual health check-ups to monitor potential conditions like non-insulin-dependent diabetes mellitus, hypertension, hypercholesterolemia, cardiovascular disease, and endometrial cancer.

 

Though PCOS-related infertility is typically manageable with fertility drugs, lifestyle modifications involving diet and exercise are fundamental for long-term management. Recent advancements have shown improvements in ovulation rates, androgen levels, pregnancy rates, and even a reduction in first-trimester miscarriage rates through the use of insulin sensitizers like Metformin to address underlying insulin resistance.

 

Most PCOS patients are young and often experience successful pregnancies with oral clomiphene or Letrozole/Femara. However, a subset of PCOS patients with severe ovarian ovulatory dysfunction and those requiring IVF treatment, will usually require injectable gonadotropin medications such as Follistim, Gonal-F, Menopur, etc. These treatments can trigger an exaggerated  response to gonadotropins, potentially leading to complications such as Severe Ovarian Hyperstimulation Syndrome (OHSS) and high-order multiple births ( triplets or greater). For these cases, employing strategies like “prolonged coasting” (see below) and/or delaying embryo transfer for a month or two  in order to allow the ovaries to recover from ovarian stimulation,  and selectively transferring fewer embryos present clear advantages..

PCOS and Egg/Embryo Quality:

 

PCOS and Egg/Embryo “Competency”.


A woman’s potential for successful egg maturation and embryo development is largely determined by genetics. However, this potential can also be significantly influenced by hormonal changes within the ovaries during the pre-ovulatory phase of her menstrual cycle. Achieving the right stimulation of the follicles and precise timing for egg maturation with the LH (Luteinizing Hormone) “surge” or through hCG (human chorionic gonadotropin) administration is crucial for optimal egg quality, fertilization, and subsequent embryo development.

 

Two key hormones, LH and FSH (follicle stimulating hormone), play vital but distinct roles in the development of eggs and follicles. FSH mainly stimulates granulosa cells (lining the follicles) and estrogen production (E2). On the other hand, LH primarily acts on the ovarian stroma (connective tissue around the follicle) to produce androgens ( predominantly testosterone and androstenedione). While a small amount of androgen supports egg and follicle development, excessive exposure can be harmful. Too much androgen can also hinder estrogen-induced growth of the uterine lining.

 

PCOS is commonly associated with elevated LH levels, leading to excess stromal growth, follicle overgrowth (referred to as cysts), and heightened androgen production. Accordingly, suppressing LH secretion using gonadotropin releasing hormone (GnRH) agonists like Lupron/ Buserelin/Superfact and decapeptyl proves beneficial. However, it is important to understand that  some LH is essential for optimal egg and follicle development. Excessive  LH on the other hand results in over-production of LH-induced ovarian androgens, which upon reaching the follicular fluid often  compromises both follicle and egg development.  Consequently, PCOS women who commonly over-produce LH and ovarian androgens  frequently propagate poorly developed follicles and  “dysmature/immature” eggs leading to  poor fertilization and embryo quality as well as an androgen-induced insufficient uterine lining that might prejudice embryo implantation, It is in my opinion, that the compromised egg quality is not necessarily due to an inherent “egg defect “ but  rather due to an adverse ovarian hormonal milieu which can often be avoided by  tailoring stimulation protocols so as to avoid excessive LH-induced androgens, Avoiding .

 

Varieties of PCOS:

 

Polycystic Ovary Syndrome (PCOS) comes in various forms, each requiring tailored treatment. Here, I wish to shed light on the main types and how infertility linked to ovulation dysfunction can be managed.

  • Hypothalamic-Pituitary-PCOS:
    • Most common form with genetic roots.
    • Characterized by high levels of Luteinizing Hormone (LH) and androgen hormones.
    • Often associated with insulin resistance.
  • Adrenal PCOS:
    • Excess male hormones come from overactive adrenal glands.
    • Elevated testosterone and/or androstenedione levels, along with increased dehydroepiandrosterone (DHEAS) levels, confirm diagnosis.
  • Pelvic Adhesive Disease-Related PCOS:
    • Linked to severe endometriosis, pelvic inflammatory disease, or extensive pelvic surgery.
    • Lower response to ovulation induction.
    • Notably, DHEAS levels remain unaffected.

 

Treating Infertility Due to Ovulation Dysfunction:

  • Hypothalamic-Pituitary-/Ovarian PCOS:
    • Successful treatment with fertility drugs like clomiphene citrate, Letrozole, or gonadotropins.
    • In-vitro Fertilization (IVF) is increasingly favored.
    • Oral Metformin can help reduce insulin resistance and androgen levels.
  • Adrenal PCOS:
    • Treated with steroids like prednisone or dexamethasone to suppress adrenal androgen production.
    • Combined with fertility drugs for induced ovulation.
  • PCOS due to Pelvic Adhesive Disease:
    • Often linked to compromised ovarian reserve and higher FSH levels.
    • Requires high doses of gonadotropins and “estrogen priming” for effective ovulation induction or IVF.

 

 

The Risks of Treatment

 

  • High-order multiple pregnancies (triplets, or greater):

PCOS patients undergoing ovulation induction are at greater risk of multiple pregnancies which are especially treacherous both mother and offspring occur with the occurrence of high-order multiple pregnancies. This risk is not preventable when ovulation induction alone is used (with or without IUI) since there is no ability to regulate the number of eggs that are ovulated. Conversely, IVF  allows for the  number of embryos transferred to the uterus to be deliberately regulated. 

 

  • Severe Ovarian Hyperstimulation (OHSS)
    1. OHSS is a significant concern for women with PCOS undergoing fertility treatments , especially where gonadotropins are administered for ovarian stimulation.
    2. Understanding OHSS:
      • Women with PCOS tend to hyper-respond to fertility drugs, often producing excessive ovarian follicles.;
      • his can escalate into OHSS, posing life-threatening risks.

 

Indicators of OHSS:

  • OHSS begins with an abundance of ovarian follicles (often more than 25).
  • Rapid rise in estradiol (E2) levels, sometimes exceeding 3000pg/ml within 7-9 days of stimulation.
  • The risk of OHSS exceeds 80% when the peak blood estradiol level exceeds 6000pg/ml.

 

Symptoms and Signs of OHSS:

 

  • Abdominal swelling due to fluid accumulation (ascites).
  • Sometimes fluid in the chest cavity (hydrothorax) and even around the heart ( pericardial effusion)
  • Rapid weight gain (more than a pound per day) due to fluid retention.
  • Abdominal pain and lower backache.
  • Nausea, diarrhea, and vomiting.
  • Visual disturbances like blurred vision and spots in front of the eyes.
  • Reduced urine output.
  • Cardiovascular complications and bleeding tendencies.

 

Managing OHSS:

 

  • If fluid accumulation compromises breathing, elevating the head of the bed often helps.
  • Drainage of excess fluid through transvaginal sterile needle aspiration (vaginal paracentesis) may be necessary.
  • Symptoms typically subside within 10-12 days of hCG shot if pregnancy doesn’t occur or by the 8th week of pregnancy.
  • Monitor urine output and perform chest X-rays and blood tests regularly to assess the condition.
  • In severe cases, hospitalization and intensive care might be necessary.

 

Avoiding OHSS while protecting egg quality though  “Prolonged Coasting”

 

In the early 1990s, I introduced  a game-changing approach to the prevention of OHSS, called “Prolonged Coasting” (PC) . The method avoids the life-endangering risks associated with this complication while to largely protecting  egg quality . PC  has now become a standard treatment for OHSS prevention. However, the effective success of PC is very largely dependent on meticulous implementation and proper timing.

 

What is “Prolonged Coasting” (PC)?

  • PC involves a strategic pause in administering gonadotropin therapy, while continuing GnRHa (Lupron/Buserelin/Superfact/decapeptyl)
  • This method significantly reduces the risk of OHSS, a life-threatening condition associated with excessive follicle growth.
  • Balancing Act for Egg Quality:
  • While PC is highly effective in averting OHSS, concerns were raised about potential impacts on fertilization rates and embryo implantation.
  • Experience suggests that the perceived egg/embryo quality deficit isn’t directly caused by PC but is more about precise timing.
  • Timing is Crucial: It is initiated when a woman with >25 follicles (total) with an estradiol measurement of >2500pg/ml has at least 50% of her follicles at 14mm diameter. It ends when the rising E2 plateaus and then drops. The key is to wait until the plasma estradiol concentration drops below 2,500 pg/ml before administering hCG. Initiating PC too early or too late can either halt follicle growth abruptly or lead to cystic follicles, both affecting egg quality. The timing allows for a progressive rise in estradiol levels followed by a plateau before a controlled decline, optimizing egg maturation. Even if the estradiol level falls below 1,000 pg/ml by hCG trigger time, resisting the urge to trigger prematurely with hCG is vital. This ensures eggs have adequate time for optimal development, increasing the chances of successful fertilization and embryo quality.

:

Words of caution:

 

  • Pituitary suppression with GnRH antagonists (Ganirelix, Cetrotide, Orgalutron) can falsely suppress E2 levels and in my opinion, is not be suitable, especially in cases like PCOS a decision on timing for PC in large part hinges on the accurate determination of serial blood estradiol levels…Accordingly, I caution against their use in patients with PCOS where “prolonged coasting is contemplated being used.
  • The standard practice of administering hCG (human chorionic gonadotropin) in an attempt to prematurely arrest further follicle growth and so prevent Severe Ovarian Hyperstimulation Syndrome (OHSS) can, by abruptly halting egg development, impact their maturation, prejudice their “competency” and in turn compromise embryo competency”, as well. Mastering the art of “Prolonged Coasting” is a critical step forward in fertility treatments. Precise timing and a patient-centered approach can make a world of difference, providing hope and improved outcomes for women on their journey towards motherhood.

 

 

In summary, when it comes to managing infertility in PCOS women, it is  crucial to tailor stimulation protocols during IVF to minimize exposure to excessive LH-induced ovarian androgens. By limiting the use clomiphene snd Letrozole/Femara  as well as LH-containing gonadotropins like Menopur and incorporating “prolonged coasting,” we can provide the necessary time for optimal follicle and egg development before administering hCG. This approach can potentially enhance egg quality and improve outcomes in IVF for women with PCOS.

_____________________________________________________________________

Herewith are  online links to 2  E-books recently  co-authored with  my partner at SFS-NY  (Drew Tortoriello MD)……. for your reading pleasure:

  1. From In Vitro Fertilization to Family: A Journey with Sher Fertility Solutions (SFS) ; https://sherfertilitysolutions.com/sher-fertility-solutions-ebook.pdf

 

  1. Recurrent Pregnancy Loss and Unexplained IVF Failure: The Immunologic Link ;https://drive.google.com/file/d/1iYKz-EkAjMqwMa1ZcufIloRdxnAfDH8L/view

 

I invite you to visit my very recently launched “Podcast”,  “HAVE A BABY” on RUMBLE;   https://rumble.com/c/c-3304480

If you are interested in having an online consultation with me, please contact my assistant, Patti Converse at 702-533-2691 or email her at concierge@sherivf.com\

 

Name: shontae s

nd can use own eggs/embryos.

 

I am a current patient. 2 failed ivf. High NK. Myself and husband have 2 variants of MTHFR I did intralipids in both. I was wondering if we would have better chances if we used a donor embryo vs using embryo we have using my husbands sperm. With that said do you recommend a donor embryo site or do you have donor embryos that are best suited for us?

Answer:

No, I do not think you need to go on Lovenox and use own eggs/embryos.

 

Hereditary Clotting Defects (Thrombophilia)

 

Geoffrey Sher MD

 

Thrombophilia (Hereditary Clotting Defect) is defined as the genetic predisposition to developing intravascular thrombosis. It is due to hypercoagulability of blood leading to impairment of initial vascularization that takes place during implantation.

Thrombophilia affects as many as one in five people in the United States and is responsible for pregnancy loss (most particularly after the 1st trimester) and “unexplained” infertility, as well as being a factor in some cases of “unexplained” IVF failure. Whether (and/or the extent to which) thrombophilia causes 1st trimester recurrent pregnancy loss (RPL) is the subject of debate and is controversial. In fact, first-trimester RPL is far more likely to be due to immunologic implantation dysfunction (IID) and/or irregularities in the contour of the uterine cavity or insufficient thickness of its lining (a thin endometrium). Thrombophilia has also been associated with late pregnancy-induced complications such as preeclampsia, premature separation of the placenta (abruptio placenta), placental insufficiency with intrauterine growth retardation, and in “unexplained” intrauterine death.

This having been said, it is a fact that most women with a thrombophilia go on to experience healthy pregnancies.

Diagnosis of Throbophilia

Thrombophilia is diagnosed when one or more of the following is detected:

  • Mutational defect involving methylenetetrahydrofolate reductase (MTHFR), which occurs in at least 20% of affected cases. Homozygosity for a common C677T mutation in the MTHFR gene that is associated with hyperhomocysteinemia is the most common form of hereditary thrombophilia leading to a 3-fold increase in risk of complications.
  • Mutation of factor V Leiden (FVL),
  • A mutation of prothrombin G20210A,
  • Deficiency of antithrombin III
  • Deficiency of protein C
  • Deficiency of protein S

Risk Factors

  • Pregnant women with predisposing factors such as:
  • A personal or family history of thromboembolism (deep vein thrombosis), pulmonary embolism (blood clot in the lung), cerebrovascular accidents (i.e. strokes)
  • A personal history of pregnancy complications such as unexplained intrauterine death, preeclampsia, abruptio placenta, intrauterine growth retardation, placental insufficiency, should be tested for the condition.

Treatment

Treatment should be initiated as soon as possible after pregnancy is diagnosed biochemically (blood or urine hCG test) and be continued throughout gestation.

Severe thrombophilias (e.g. homozygous MTHFR mutations, protein C deficiency, prothrombin G20210A mutation) as well as cases of mild thrombophilias associated  with one or more of the pregnancy complications mentioned above, are best treated with low-molecular weight heparin (LMWH) taken throughout pregnancy.

For other (milder) thrombophilias and no history of prior pregnancy complications: Low-dose aspirin with the B vitamins folic acid, B6 and B12.

_________________________________________________________________________________________________________________

Herewith are  online links to 2  E-books recently  co-authored with  my partner at SFS-NY  (Drew Tortoriello MD)……. for your reading pleasure:

  1. From In Vitro Fertilization to Family: A Journey with Sher Fertility Solutions (SFS) ; https://sherfertilitysolutions.com/sher-fertility-solutions-ebook.pdf

 

  1. Recurrent Pregnancy Loss and Unexplained IVF Failure: The Immunologic Link ;https://drive.google.com/file/d/1iYKz-EkAjMqwMa1ZcufIloRdxnAfDH8L/view

 

I invite you to visit my very recently launched “Podcast”,  “HAVE A BABY” on RUMBLE;   https://rumble.com/c/c-3304480